MPRE Study Questions pt. 2

Lakukan tugas rumah & ujian kamu dengan baik sekarang menggunakan Quizwiz!

Attorney Ayers limits her practice to family law. Married couple Hector and Wanda came to her, hoping to save their marriage. After hearing their story, Ayers explained that she could act as a third-party neutral between them—not representing either one of them, but facilitating their efforts to work through their difficulties. Ayers explained that neither of them would have the protections afforded by an ordinary attorney-client relationship, such as the attorney-client privilege, and both said that they understood. Ayers held a series of meetings with the couple (sometimes with both, and other times with just one). Ayers began each meeting with a reminder that, in the event of later litigation, between Hector and Wanda, the privilege would not protect what was said at the meeting. At one of Ayer's individual meetings with Hector, he disclosed that he "occasionally" uses cocaine and that he "sometimes" uses the family's savings to buy the drug. Ultimately, Hector and Wanda were unable to resolve their differences, and Wanda sued Hector for a divorce and for custody of their three children. At the custody hearing, Wanda's lawyer called Ayers to the witness stand and asked: "What, if anything, did Hector tell you about his use of cocaine?" Ayers refused to answer, citing the attorney-client privilege. Which of the following is correct? (A) The privilege claim should be overruled; ifAyers refuses to answer, she would be subject to litigation sanction. (B) The privilege claim should be sustained; ifAyers reveals what Hector said, she would be subject to discipline. (C) Ayers is subject to discipline for attempting to serve as a third-party neutral in a family law matter.(D) Ayers is subject to civil liability for attempting to serve as a third-party neutral in a family law matter.

(A) (C) is incorrect because there is no disciplinary rule against a lawyer acting as a third-party neutral in a family law matter, and it was proper for Ayers to undertake the neutral's role in helping Hector andWanda try to resolve their marital problems. [See Restatement § 130, comment (D) is incorrect because the facts contain nothing to support a civil claim againstAyers for the work she did as a third-party neutral. (B) is incorrect because when two parties jointly consult an attorney on a matter of common interest, neither of them can claim the attorney-client privilege in subsequent civil litigation between them.[See Restatement §75] (A) is correct because Hector cannot claim the privilege for the reason stated above. If the judge overrules the privilege claim and Ayers nevertheless refuses to answer the question, she can be sanctioned for contempt of court.

Client Crowell made a preliminary contact with lawyer Lear to see if she wanted to hire Lear to defend her in a tort case that had been assigned to Judge Johnson. Lear told her that the initial consultation was free of charge. After listening to Crowell's brief outline of the case, Lear told her: I know how to get a favorable decision from Judge Johnson. He will be running for reelection 18 months from now, and he will need money for his campaign. You should send him a $2,000 campaign contribution now, with a nice note wishing him well in his bid for reelection. Johnson's opponent in the election will be a local lawyer, Willard Wampler. Wampler is an honest fellow, but I know that his two brothers are associated with organized crime. I can write a guest editorial for the local paper, praising Johnson's .judicial record and implying that Wampler is a crook. With your contribution and my letter, I think we can count on Judge Johnson to reach a wise decision in your case. Crowell hired Lear and sent Judge Johnson the $2,000. Lear wrote the guest editorial, and it was published in the local paper. For which of the following is Lear subject to discipline? I. Saying that he knew how to get a favorable ruling from Judge Johnson. II. Advising Crowell to send Judge Johnson a campaign contribution. III. Writing the guest editorial. IV. Accepting the case following free legal advice. (A) I., II., and III. only. (B) I., II., III., and IV. (C) II. only. (D) I., II., and IV. only.

(A) A lawyer must not state or imply that he has the ability to improperly influence a government official or that he can achieve results by means that violate the Rules ofProfessional Conduct or other law. [ABAModel Rule 8.4(e)] Thus, Lear is subject to discipline for saying that he knows how to get a favorable ruling from Judge Johnson, which implies that he can influence the judge. Advising Crowell to make the campaign contribution can be viewed in two ways, both of which subject Lear to discipline. If the $2,000 is viewed harshly as a bald bribe, then Lear is subject to discipline for counseling a client to commit a crime. [ABA Model Rule 1.2(d)] If the$2,000 is viewed benignly as a campaign contribution, Lear is subject to discipline for assisting a judge to violate a judicial ethics rule. [ABA Model Rule 8.4(0] TheCode of Judicial Conduct prohibits JudgeJohnson from personally accepting campaign contributions, and it prohibits his campaign committee from accepting contributions more than a year before the election. [CJC 5C(2)] Thus, item II. subjects Lear to discipline. Lear is subject to discipline for item III. because Lear apparently believes that Wampler is an "honest fellow," yet his editorial implies that Wampler is a crook. A lawyer is prohibited from making a statement about a judicial candidate, either knowing that the statement is false or with reckless disregard as to its truth or falsity. [ABA Model Rule8.2(a)] Lear is not subject to discipline for item IV. because he was not offering unsolicited free legal advice, then accept-ing employment arising from it. [See ABAModel Rule 7.3] Crowell initiated the contact, asked for the advice, and was in the midst of negotiating Lear's employment when the advice was given.

Solo practitioner Sololei limits her practice to two kinds of cases. First, she represents plaintiffs who have been physically injured by medical malpractice. Second, she represents plaintiffs who have been physically injured by household products that have turned out to be harmful to health. When Sololei turned67, she began looking for someone to buy her law practice. Ultimately, she sold the medical malpractice part of her practice to Lawyer Lang for $400,00, and she sold the household products part of her practice to attorney Abel for $250,000. Sololei invested the sale proceeds in high-yield stocks and bonds, and she moved to Fiji to sit on the beach, sip refreshing fruit drinks, and enjoy her retirement. Unfortunately for Lang and Abel, within 30 days after the sales, approximately 40% of the clients they received from Sololei decided to collect their case files and take their business to different lawyers. Were the sale from Sololei to Lang and the sale from Sololei to Abel proper? (A) Yes, even though Sololei sold pieces of her practice to two different buyers, and even though 40% of Sololei's clients left the buyers within 30 days. (B) Yes, but Sololei is subject to civil liability to the two buyers for unjust enrichment because 40% of Sololei's clients left the buyers within 30 days. (C) No, because Sololei sold pieces of her practice to two different buyers. (D) No, because 40% of Sololei's clients left the buyers within 30 days.

(A) ABA Model Rule 1.17 permits a lawyer to sell her entire law practice, or an area of her law practice, to one or more lawyers or law firms. Here, Sololei sold her entire law practice to two different lawyers, and ABAModel Rule 1.17 permits that. The departure of 40% of Sololei's clients does not cause the sales to violate ABA Model Rule1.17. Indeed ABA Model Rule 1.17(c)(2)requires the selling lawyer to notify her clients that they have a right to pick up their files from the buyer and take them to a different lawyer. [See also Comment 2 toABA Model Rule 1.17] (B) and (D) are wrong because clients are not like sheep that can be bought and sold, willy-nilly. To hold Sololei either subject to discipline or civilly liable in unjust enrichment would be inconsistent with the clients' right to pickup their files and take them to a different lawyer. Sololei, Lang, and Abel made their sales contracts in the context of ABAModel Rule 1.17 so Lang and Abel cannot claim that they were surprised when clients departed. (C) is wrong because ABAModel Rule 1.17(b) permits a lawyer to sell her entire practice to one or more lawyers or law firms. One might also argue that Sololei's medical malpractice cases are in a different "area of practice" (professional malpractice) from her household products cases (products liability). But that argument is not necessary here because Sololei sold her entire law practice.

Probate attorney Adamson was representing the executor of decedent Denman's estate. The executor removed the contents of Denman's safe deposit box and brought them to Adamson to be inventoried and appraised. The items included Denman's collection of valuable antique gold coins. Adamson put the coin collection into a heavy brown envelope and labeled it as part of the Denman estate. Because he intended to start preparing the inventory immediately after lunch, Adamson put the brown envelope and Denman's other belongings into the file drawer of his desk; he then left for lunch without locking the file drawer. Adamson's secretary saw the coins and saw what Adamson did with them. During the lunch hour, the secretary took the envelope of coins and disappeared, never to be seen again. Is Adamson subject to discipline? (A) Yes, because he did not put the coins in a safe place. (B) Yes, because he is responsible for his employee's dishonest act. (C) No, because the loss was proximately caused by the secretary's dishonesty, not by his conduct. (D) No, because he took reasonable precautions to safeguard the coins in the circumstances.

(A) Adamson is subject to discipline because he did not put the coins in a safe place.When a lawyer comes into possession of property to be held on a client's behalf, the lawyer must identify it as belonging to the client and must put it in a safe place. [ABAModel Rule 1.15(a)] Although the Rules do not define "safe," common sense suggests that allowing a valuable coin collection to be viewed by employees, placing it into an unlocked desk file, and then leaving the office is not safe. A lawyer should use the same level of care required of professional fiduciaries. Adamson's actions fall well short of that. (B) is wrong. Adamson maybe liable to the estate in civil damages for his secretary's dishonest act, but the question here is professional discipline, not civil liability. Adamson could be disciplined if he did not take reasonable steps to train his secretary properly [ABA ModelRule 5.3], but if he took such steps, he should not be disciplined for her criminal act.(C) is wrong. The issue here is Adamson's failure to safeguard the coins; the proximate cause of the loss is beside the point.Technically, Adamson could be subject to discipline for failure to safeguard the property even if no loss occurred. (D) is wrong. Placing the coins in an unlocked desk file and leaving the office was not a reasonable way to safeguard them.

In an effort to prevent homosexuals from moving to Elmville, the Elmville city council passed a zoning ordinance that prohibits the use of any dwelling house as a residence for two or more adults of the same sex unless they are related to each other. Violation of the ordinance is a misdemeanor and carries a fine of $10,000. Carlo owns several rental houses in Elmville, and he was outraged when the City Council passed the ordinance. He sought the legal advice of attorney Ahmad. Ahmad advised him that the ordinance could probably be overturned as a violation of rights guaranteed by the state constitution, but that one would have to obtain legal standing to challenge the ordinance. Ahmad advised that one way to obtain legal standing would be for a landlord to bring an appropriate action for declaratory judgment, and another way would be to simply violate the ordinance and raise the constitutional challenge as a defense to its enforcement. After receiving this advice, Carlo promptly rented one of his houses to two gay men. Carlo was cited for violation of the ordinance. Was Ahmad's conduct proper? (A) Yes, because violating the ordinance was one of the few ways to gain legal standing to challenge the constitutionality of the law. (B) Yes, if the ordinance is ultimately held unconstitutional. (C) No, because Ahmad counseled and assisted Carlo in conduct Ahmad knew was illegal.(D) No, because Ahmad did not advise against renting houses to unrelated adults of the same sex.

(A) Ahmad's conduct was proper because violating the ordinance was one of the ways to gain legal standing to challenge the constitutionality of the ordinance. A lawyer must not counsel or assist a client in con-duct that the lawyer knows is criminal or fraudulent. However, a lawyer may counselor assist a client to make a good faith effort to determine the validity, scope, meaning, or application of a law even if it requires the client to disobey the law. [ABA ModelRule 1.2(d)] This usually arises when a lawyer is asked how a particular law maybe challenged, and the lawyer advises the client on ways to obtain legal standing, which includes disobedience of the law. Here, Ahmad was merely advising Carlo on methods of obtaining legal standing, including renting houses in violation of the ordinance. (B) is wrong because the ultimate outcome of the challenge is irrelevant. It is important that Ahmad believed there was a good faith basis for challenging the validity of the ordinance, but whether the challenge succeeds does not determine the propriety of his conduct. (C) is wrong because, as discussed above, there is an exception to this rule for a good faith effort to determine the validity of a law. (D) is wrong because there is no affirmative duty to counsel the client in this way. In fact, as discussed above, there is an exception for a good faith challenge to the law that would permit Ahmad to do just the opposite.

Attorney Anderson is licensed to practice in State Red, but he is not engaged in the active practice of law. Anderson and Benson, a nonattorney partner, operate a temporary placement service lot legal secretaries in State Green, which borders State Red. Anderson is not licensed to practice law in State Green, nor does he hold himself out to be so licensed. An investigation by State Green authorities results in the discovery that Anderson and his partner have intentionally filed improper state business tax returns. Is Anderson subject to discipline in State Red? (A) Yes, because his actions in State Green constitute fraud. (B) Yes, if he supervises the temporary service business from State Red. (C) No, because Anderson is not licensed to practice law in State Green. (D) No, because this situation does not involve the practice of law.

(A) Anderson is subject to discipline because his actions in State Green constitute fraud.A lawyer is subject to discipline not only for violating a disciplinary rule, but also for committing a criminal act that reflects adversely on his honesty, trustworthiness, or fitness as a lawyer in other respects, or for engaging in conduct involving dishonesty, fraud, deceit, or misrepresentation.[ABA Model Rule 8.4] The type of fraud described clearly falls within this rule. (B) is wrong because a lawyer is subject to discipline in a state where he is licensed to practice even if the misconduct occurred in another jurisdiction. [ABA Model Rule8.5(a)] (C) is wrong because the fact thatAnderson was not licensed in State Green is irrelevant. He was not practicing law in State Green, so he is not subject to discipline on the ground of unauthorized practice. He is subject to discipline, however, for filing improper tax returns. As discussed above, Anderson cannot escape discipline for his fraudulent conduct because it occurred in another jurisdiction. (D) is wrong because, as discussed above, a lawyer may be disciplined for dishonest conduct regard-less of whether it is related to the practice of law.

Paralegal Platen works for the law firm of Dahlers & Sentz. Her direct supervisor is partner Dahlers, whose practice is limited to international trade law. Partner Sentz is the firm's leading trial lawyer, both in commercial and personal injury cases. On her way to work one morning, Platten saw a pedestrian run down in a crosswalk by a speeding car. Platten rendered first aid, and while she was waiting with the pedestrian for the ambulance, Plattcn said: "Here, call the number on this card and talk to attorney Seymour Sentz; he's really good, and he can help you recover money for the injury you have suffered." When she got to work, she told partner Dahlers what she had done. Dahlers admonished Platten not to hand out the firm's cards in such situations, but he did not discuss the matter with partner Sentz. Is Dahlers subject to discipline? (A) Yes, because he failed to warn Sentz not to take the pedestrian's case. (B) Yes, because as Platten's supervisor, he is responsible for any unethical act she commits. (C) No, because as a nonlawyer, Platten is free to recommend a lawyer to someone if she wishes. (D) No, because Platten may not have been aware at the time that she did anything wrong.

(A) Dahlers is subject to discipline for failing to warn Sentz not to take the case. If Platten were a lawyer, her conduct would violate ABA Model Rule 7.3(a), which prohibits in-person solicitation. The partners in a firm are responsible for educating their nonlawyer employees about ethics issues and making reasonable efforts to assure that those employees comply with ethics rules. [ABA Model Rule 5.3(a)]Moreover, a partner is subject to discipline if he learns about the violation of an ethics rule by a nonlawyer employee "when its consequences can be avoided or mitigated," but the partner "fails to take reason-able remedial action." In this case, the consequences of Platten's solicitation could have been avoided by warning Sentz not to take the pedestrian's case. Because he failed to warn Sentz, Dahlers is subject to discipline. (B) is wrong because it is too broad. A lawyer's responsibility for a nonlawyer employee's ethics violation is limited to situations in which the lawyer orders it, ratifies it, or learns about it in time to remedy it and does not do so. [ABAModel Rule 5.3(c)] (C) is wrong because even though people are generally free to recommend a lawyer to someone else, that does not allow paralegal Platten to solicit business for the firm that employs her.[ABA Model Rule 8.4(a)] (D) is wrong because Dahlers and the other partners inthe firm had a duty to educate Platten about ethics rules. [ABA Model Rule 5.3(a)]Furthermore, even if Platten acted innocently, that does not excuse Dahlers's failure to warn Sentz not to take the case.

Defendant Dennis asked lawyer Liston to defend him in a criminal case in which Dennis was charged with running a gambling operation. Dennis was known in the community as a wealthy person, but one who seldom kept his word and seldom paid his bills. Liston agreed to do the necessary work for a fiat fee of $5,000, which was reasonable in light of the difficulty of the case and the number of hours required. However, Liston required that Dennis pay $3,000 in advance. When Dennis protested that he did not have that amount available in ready cash, Liston accepted Dennis's full-length fur coat in lieu of the cash advance. The fair market value of the coat is $3,000, and Liston agreed to return it upon payment of his fee. Their agreement was reduced to writing and signed by both parties. After Liston had put in considerable time in preparing the case for trial, Dennis fired Liston for no good reason and refused to pay him anything for the work already done. Assuming the reasonable value of Liston's services to date is $4,000, which of the following statements is correct? I. Liston is subject to discipline for demanding that Dennis pay $3,000 in advance, before any legal services had been rendered. II. Liston is subject to discipline for taking the coat in lieu of cash as an advance on legal fees. III. Liston must promptly return the coat to Dennis upon termination of their relationship, and submit a bill for his services. IV. If Liston returns the coat and sues Dennis to collect his fee, Liston will be entitled to recover the full contract price ($5,000) because Dennis terminated the contract without good cause. (A) None of the above. (B) I., II., and III. only. (C) IV. only. (D) I., III., and IV. only.

(A) Item I. is wrong because a lawyer may require his fee to be paid in advance.[Comment 4 to ABA Model Rule 1.5] ItemII. is wrong because a lawyer may accept property in return for services, provided it does not involve a proprietary interest in the subject of litigation. [Comment 4 toABA Model Rule 1.5] Here the coat is not the subject of litigation; thus, accepting the coat as payment was proper. Item III. is wrong because (i) as noted above, it is acceptable to take property in payment for services, and (ii) Liston has earned the fee represented by the coat. When a lawyer-client relationship is prematurely terminated, the lawyer is required to return to the client any unearned advance on attorneys' fees. [ABA Model Rule 1.16(d)] In this case, at the time he was fired, Liston had earned $4,000 in fees. Because the coat is worth only $3,000, not only may Liston keep the coat, he may sue Dennis for the$1,000 difference. Item IV. is wrong because in the event the relationship is prematurely terminated, the lawyer is generally entitled only to fees actually earned under a quantum meruit theory of recovery. Thus, in this case, Liston could recover $4,000.

Lawyer Lacey is on the in-house legal staff of Transcorp, Inc. In that capacity, she works daily with Transcorp's top executive officers. She was assigned to defend Transcorp in a lawsuit brought by West America Bank to collect a $750,000 promissory note. The note was signed on behalf' of Transcorp by Willard Westerman, Transcorp's treasurer and chief financial officer. Transcorp's defense is that Westerman had no authority to sign the note and that the bank knew it. Transcorp has advised Westernlan that it may seek indemnification from him if it is held liable to the bank. Westerman is not represented by counsel. Shortly before Westerman was to have his deposition taken by the bank, Westerman called Lacey and asked her what to expect at the deposition and how to respond to the bank's questions. What should Lacey do? (A) Not discuss the matter with Westerman, and, if appropriate, advise him to hire a lawyer to represent him at the deposition. (B) Tell Westerman that she cannot discuss the matter with him unless he wants her to represent him at the deposition. (C) Advise Westerman to tell the truth, to answer fully all questions that are asked, and to pause before each answer to give her time to object to the bank's questions. (D) Advise Westerman that his own interests will be best served by answering truthfully and demonstrating, if he can, that he had authority to sign the note.

(A) Lacey should not discuss the matter with Westerman, and should advise him to hire his own attorney. When an organization is the lawyer's client, the lawyer owes the duty of loyalty to the organization—not to the people who are its constituents. If there is a conflict between the interests of the organization and the interests of one of its constituents, the lawyer should advise the constituent to obtain independent legal counsel. [Comment 10 to ABA Model Rule1.13] Lacey's client is Transcorp, and Transcorp's interests are in conflict with the interests of Westerman. If Transcorp proves that Westerman had no authority to sign the note, the bank may sue Westerman himself. If Transcorp is held liable to the bank, Transcorp may sue Westerman for indemnification. In these circumstances, Westerman needs his own lawyer, and Lacey must not try to represent both him and Transcorp. Furthermore, because Westerman is presently unrepresented in the matter and his interests conflict with those of Transcorp, Lacey must not try to give him legal advice, except to get a lawyer. [See ABA Model Rule 4.3] (B)is wrong because Lacey must not try to represent Westerman at the deposition due to the conflict of interest explained above. (C) is wrong because Lacey must not try to give Westerman legal advice, except to get a lawyer. (D) is wrong for the reason just stated. Furthermore, as Transcorp's lawyer,Lacey must not give legal advice to a person with conflicting interests (particularly advice to testify in such a way as to damage her client's position).

Lawyer LeBrille is admitted to practice in State A. One of her regular clients is Chatsworth Inc., which is incorporated in and has its principal place of business in State A. The president of Chatsworth went to France to negotiate a business contract for Chatsworth that would be governed, in part, by the law of the European Community ("EC"). The president telephoned LeBrille to ask whether a particular provision of the proposed contract would be lawful under EC law. The president needed a quick answer because he had to resume the contract negotiation a few minutes later. LeBrille had studied EC law, but she was not admitted to practice in any nation that is a member of the EC. LeBrille warned the president about the danger of relying on off-the-cuff, unresearched legal advice, but he asked her to do the best she could. She then advised him that the contract provision would be lawful under EC law. The president thanked her, continued the contract negotiation, and signed a contract for Chatsworth that included the questioned provision. As it turned out, LeBrille's advice was mistaken: the provision violated EC law and rendered the contract unenforceable. Is LeBrille subject to discipline? (A) No, because she did the best she could in an emergency situation. (B) No, because a State A lawyer is not expected to be competent in EU law. (C) Yes, because she is not licensed to practiceEU law. (D) Yes, because she gave legal advice without adequate research.

(A) LeBrille is not subject to discipline because she did the best she could in an emergency situation. In an emergency situation, a lawyer may give legal advice on a matter that she would not be competent to handle in an ordinary situation. [Comment 3 toABA Model Rule 1.1] Here, LeBrille's advice was limited to the narrow question the client posed. She is not subject to discipline because she did the best she could in the heat of the moment, and she warned the client about the dangers of relying on un-researched legal advice. (B) is wrong because a State A lawyer who renders advice about the law of another jurisdiction in an ordinary, non emergency situation is expected to be competent to render such advice. [ABA Model Rule 1.1](C) is wrong because LeBrille is not en-gaged in unauthorized practice when she advises a State A client about EU law. She need not be admitted in an EU member nation in order to advise her State A client about EU law. (D) is wrong because LeBrille is not subject to discipline for rendering legal advice that turns out to be mistaken if she acted competently in light of the time-constrained circumstances in which she found herself.

When lawyer Locke was an associate in the firm of Bliss & Buford, she did the legal work for one of the firm's clients, Cannon, on a land sale transaction that earned Cannon millions of dollars. In gratitude, Cannon asked Locke whether she had any unfulfilled wishes. Locke told him that she wished she had enough money to start her own solo ]aw practice. Cannon then told her that he would lend her $100,000 to set up her new practice. In return, she would thereafter do all of his legal work at a 5% discount from her normal hourly fee, and she would pay Cannon 10% of the legal fees she earned in solo practice until the $100,000 loan was fully repaid. Locke was delighted. She drafted a complete, detailed agreement between herself and Cannon, and she insisted that he get outside legal advice before signing the agreement. Cannon got the outside advice and signed the agreement, and Locke set up her solo practice accordingly. Is Locke subject to discipline? (A) No, unless she allows Cannon to interfere with her professional judgment in handling work for other clients. (B) No, unless Locke fails to give Bliss &Buford timely notice of the transaction. (C) Yes, unless Bliss & Buford consented to the loss of Cannon as a firm client. (D) Yes, unless Cannon is a lawyer.

(A) Locke is not subject to discipline unless she allows Cannon to interfere with her judgment in handling other clients' matters.[See ABA Model Rule 5.4] (B) is wrong because there is no law or disciplinary rule that requires Locke to notify Bliss &Buford regarding the transaction. (C) is wrong because no law or disciplinary rule requires Bliss & Buford's consent toCannon's leaving the firm and giving his business to Locke as a solo practitioner.(D) is wrong because the loan payback clause does not violate the rule against splitting a legal fee with a non lawyer.[ABA Model Rule 5.4(a)] True, the clause does measure the monthly payments as a percentage of Locke's net income in the prior month and most of her net income will probably come from legal fees. How-ever, it makes sense to tailor her loan payments to her income, and the arrangement does not invite the evil that the no splitting rule was designed to prevent interference with the lawyer's professional judgment.

L'Etoille Women's Wear, Inc. is incorporated in and has its principal place of business in State A, and it is represented by a lawyer who is licensed to practice in State A. L'Etoille owns the valuable, federally registered trademark L'Etoille for use on various items of women's wearing apparel, and it licenses the trademark to garment makers in various states for use on items of apparel that are manufactured in accordance with style and quality specifications set by L'Etoille. Lawyer Londrell is licensed to practice only in State B. She represents DonnaDell Clothing Co., which is incorporated in and has its sole place of business in State B. Londrell and the president of DonnaDell traveled to State A, where they negotiated with L'Etoille's lawyer for a license to use its trademark on apparel to be manufactured at DonnaDell's plant in State B. Londrell is not licensed to practice before the United States Patent and Trademark Office. The license agreement between L'Etoille and DonnaDell provides that the agreement is to be construed in accordance with the law of State A, and that any disputes arising under the license will be arbitrated in State A. Is Londrell subject to discipline? (A) No, because admission to practice in State A was not necessary to negotiate the trademark license there. (B) No, unless the licensing agreement was actually drafted by Londrell in State A. (C) Yes, because she was not admitted to practice in State A. (D) Yes, because she was not admitted to practice before the United States Patent and Trademark Office.

(A) Londrell is not subject to discipline be-cause admission to practice in State A is not required to negotiate a licensing agreement there. A lawyer must not practice law in a state where she is not admitted to practice. [ABA Model Rule 5.5(a)] No state, however, would regard Londrell's conduct as unauthorized practice. Londrell was admitted in State B, she was represent-ing a State B client, and the trademark license has an important nexus to State B because DonnaDell's manufacturing operations will take place in State B. [SeeHazard & Hodes, §46.6] Moreover, ABAModel Rule 5.5(c)(4) permits a lawyer to temporarily practice out of state if that practice is reasonably related to the lawyer's home-state practice. Here, Londrell's going to State A to negotiate the licensing agreement was reasonably related to her State B practice of representing Donna Dell. (B) is wrong because where the agreement is actually drafted or executed is not relevant. Londrell would not be engaging in unauthorized practice in any event. (C) is wrong because Londrell need not be admitted inState A to negotiate with a company located there. (D) is wrong because a lawyer need not be admitted to practice before the United States Patent and Trade-mark Office in order to negotiate a trade-mark license. One must be admitted to practice before that agency to prosecute an application for a United States patent, but Londrell is obviously not doing that here.

Lawyer Lubovich is one of nine lawyers who practice probate law in Chichachepi County. In that county, all probate matters go before a single judge, not surprisingly called the Probate Judge of Chichachepi County. The probate judge's duties include appointing counsel for the administrators of intestate estates—serving as an administrator's counsel can be very lucrative if the estate is large and complicated. The incumbent probate judge recently retired. Her custom was to appoint out-of-county lawyers to serve as administrators' counsel; she believed that such lawyers are less subject to local political and social pressures than Chihachepi law-yers. The probate judgeship will be filled in six months in a partisan, contested election, and lawyer Lubovich is one of the candidates. Lubovich met jointly with Chihachepi County's eight other probate lawyers and said: "My friends, I welcome your support and assistance in my quest for the probate judgeship. I will be making some important changes, sucha s appointing only local lawyers as counsel for administrators of intestate estates; I see no reason to send that work out of the county. To win, I need your financial support and your public support, walking precincts, handing out brochures, talking up my qualifications to the voters—whatever you can do to help. If you'd like to contribute money or time, please get in touch with my campaign committee atwww.lubovich4judge.org." Five of the eight lawyers sent generous monetary donations to the Lubovich campaign committee for the self-confessed purpose of securing legal appointments if Lubovich wins. The other three lawyers volunteered generous amounts of their time in the Lubovich campaign, but their motives for doing so were unexpressed and unclear. Which of the following is correct?(A) Lubovich is subject to discipline for personally soliciting money and publicly stated support from the eight lawyers. The five lawyers who contributed money will be subject to discipline if Lubovich win sand if they accept appointments from him. (B) Lubovich's conduct was proper because he was a candidate for a judicial office in a contested election. All eight lawyers, however, are subject to discipline because a practicing lawyer must not contribute either money or time to the political campaign of a judge before whom the lawyer expects to appear. (C) Lubovich is subject to discipline for promising to appoint only local lawyers as counsel for administrators, in the hope of securing the publicly stated support of the eight lawyers. The conduct of all eight lawyers, however, was proper because lawyers are permitted to support or oppose candidates in contested elections for judgeships. (D) Lubovich's conduct was proper because he was a lawyer-candidate, not a judge, at the time he met with the eight lawyers. The conduct of the five lawyers who contributed money was proper because a person's motive for making a political contribution is a private matter that is protected by the penumbra of the First and Fourteenth Amendments.

(A) Lubovich is subject to discipline underCJC 5(C)(2), which prohibits a judicial candidate from personally soliciting campaign contributions or publicly stated support. The five lawyers who contributed money will be subject to discipline if Lubovich wins and if they accept appointments from him. [See ABA Model Rule7.6] (B) is wrong because Lubovich is subject to discipline under CJC 5(C)(2), as noted above. The second sentence of (B) is wrong because it overstates the constraints on a lawyer's participation in a judicial campaign. (C) is wrong because the five lawyers who contributed money violatedABA Model Rule 7.6, as noted above. The first sentence of (C) is a misapplication ofCJC 5(A)(3)(d)(i), which prohibits a judge from making pledges or promises that are inconsistent with the impartial performance of the judge's adjudicative duties "with respect to cases, controversies, or issues that are likely to come before the court." Here, the policy on whom the AttorneyGeneral will appoint as administrators' counsel is not the kind of litigation issue to which the highlighted language of CJC5(A)(3)(d)(i), above, refers. The second sentence of (C) is wrong because the motive of the five money contributors was to obtain appointments if Lubovich won. [SeeABA Model Rule 7.6] (D) is wrong be-cause a lawyer who runs for a judicial post must follow the CJC. [See CJC 5(A)(3)(a) -(c)] The second sentence of (D) is wrong because ABA Model Rule 7.6 trumps whatever privacy rights one can find in the peripheral glow of the First and Fourteenth Amendments.

The M&S law firm represented United Oil Company in a merger transaction in which United acquired all the assets of Mogul Petroleum Inc. in exchange for a specified amount of capital stock of United. M & S's work for United was limited to the antitrust and securities law issues raised by the merger, and the M & S lawyers who worked on the matter did not become privy to any confidential information concerning the routine operations of United's business. The merger was completed two years ago, and M & S has not subsequently represented United in any other matter. Recently, M & S took in a new partner, lawyer Lurner, who had previously practiced asa solo practitioner. One of the cases that Lurner brought to M & S from his solo practice was Droan v. United Oil Co., an employment dis-crimination case in which Lurner's client, Droan, claims that United fired him solely because of his age. When Lurner became an M& S partner, United promptly made a motion in the trial court to disqualify Lurner and M & S as counsel for Droan due to M & S's earlier representation of United in the Mogul merger matter. Are Lurner and M & S subject to disqualification? (A) No, because the merger matter and the discrimination case are unrelated matters and because M & S did not gain confidential information from United that would be material in the discrimination case. (B) No, because whatever material confidential information M & S might have picked up in the merger matter is not imputed to Lurner. (C) Yes, because M & S owes continuing duties of loyalty and confidentiality to its former client, United. (D) Yes, even though the merger matter and the discrimination case are unrelated and even though M & S did not gain confidential information from United that would be material in the discrimination case.

(A) Lurner and M & S are not subject to dis-qualification because from the facts given in the question, it appears that the merger matter and the age discrimination case are not substantially related to one another, and that the M & S lawyers did not gain confidential information in the merger matter that would be material to the discrimination case. [See ABA Model Rule 1.9] Therefore(C) and (D) are incorrect. (B) is incorrect because if the M & S lawyers who worked on the merger matter had obtained confidential information that would be material in the discrimination case, their knowledge would be imputed to Lurner, even though he was not a member of M & S when the knowledge was obtained. [See ABA ModelRules 1.9, 1.10]

Intron, Inc. makes computer chips. It is incorporated in State One; its headquarters and principal place of business are in State Two, and it has chip manufacturing plants in States Three and Four. Lawyer Linz is admitted to practice in State Two. He is a domiciliary and resident of State Two, and he is employed full-time as an employment discrimination lawyer in Intron's in-house law department in State Two. Intron is his only client. Intron's General Counsel—Linz's supervisor in the in-house law department—instructed him to move temporarily to State Four, where the Intron manufacturing plant was afflicted with a rash of employment discrimination claims. The General Counsel told Linz to settle all existing meritorious claims, to prepare all non meritorious claims for trial, and to train the managers of the State Four plant to comply with federal and state employment discrimination laws. The General Counsel knows that Linz is not admitted to practice in State Four. May Linz do as the General Counsel has instructed? (A) Yes, because Linz's right to practice temporarily in State Four is a debatable issue of legal ethics, and it is appropriate for such an issue to be decided by a lawyer's supervisor. (B) Yes, but if State Four requires out-of-state lawyers to be admitted pro hac vice in order to engage in pretrial preparations, Linz must seek such admission. (C) No, unless Linz takes and passes State Four's short-form bar exam for out-of-state lawyers within six months of his arrival in State Four. (D) No, unless Linz associates a State Four lawyer who will actively participate with Linz in settling the meritorious claims and preparing the non meritorious claims for trial.

(B) ABA Model Rule 5.5(c) concerns temporary practice in a state where the lawyer is not admitted. ABA Model Rule 5.5(c)(4)permits Linz to practice temporarily inState Four to the extent that his work in that state is reasonably related to the work he does for Intron in State Two. If StateFour requires out-of-state lawyers to seek pro hac vice admission before engaging in pretrial preparation, then ABA Model Rule5.5(c)(2) and (3) would require Linz to seek such admission. (A) is wrong because the application of ABA Model Rule 5.5(c)to Linz's situation is clear, not debatable. (C) is wrong because ABA Model Rule5.5(c) allows Linz to perform the assigned tasks without taking a bar exam in State Four. (D) is wrong because associating a local lawyer is only one of four different ways to satisfy ABA Model Rule 5.5(c).

Farmer Farner asked attorney Anderson to represent him in an eminent domain proceeding in which the state sought to obtain a right-of way across Farner's farm. Anderson had not handled an eminent domain case before, but she planned to make herself competent through diligent research and study. As it turned out, Anderson did not have enough time to do what she had planned, so she associated an eminent domain specialist, lawyer Ling, as her cocounsel in the case. Anderson did not consult Farner about associating Ling. Ling did about 90% of the work in the case, and Anderson did the other 10%. Together they secured a very favorable result for Farner, and Anderson sent Farner a fee bill for a reasonable amount. Farner paid the bill, and Anderson remitted 90% of the proceeds to Ling. Is Anderson subject to discipline? (A) Yes, because she took on a case she was not competent to handle. (B) Yes, because she did not consult Farner about associating Ling. (C) No, because the fee split was in proportion to the work done by the two lawyers. (D) No, because she associated a co-counsel who was competent to handle the case.

(B) Anderson is subject to discipline because she did not consult Farner about associating Ling. A lawyer may split her fee with a lawyer outside her firm if the total fee is reasonable, the split is either in proportion to the work done (or in some other proportion if the splitting lawyers assume joint responsibility), and the client agrees to the split in a writing that discloses the share that each lawyer will receive. [ABA ModelRule 1.5(e)] Because Anderson did not tell Farner about the arrangement, she is subject to discipline. (A) is wrong because a lawyer may take on a case she knows she is not competent to handle if she does the study needed to become competent, or if she associates a lawyer who is competent to handle it. [Comment 1 to ABA Model Rule1.1] (C) and (D) are wrong because they ignore the need to inform Farner.

Attorney Aronson has just opened an office in a town where he does not know many people and has few contacts. Aronson is attempting to build up a clientele for his general civil and criminal law practice. Aronson has just heard on the radio that a group of several hundred townspeople have been arrested and are being held at the county jail for conducting a noisy demonstration outside the local high school to protest an impending strike by the teachers. The radio reports that the townspeople are charged with unlawful assembly, resisting arrest, and disorderly conduct. Aronson believes that the arrests of these demonstrators is a politically motivated trick, and that the demonstrators have been deprived of their freedom of expression in violation of the First Amendment. Aronson goes down to the county jail and offers his legal services, free of charge, to any of the arrested demonstrators who want legal assistance. Are Aronson's actions proper? (A) Yes, unless he is motivated in any way by a desire to obtain publicity for his law practice. (B) Yes, because Aronson was offering his services free of charge. (C) No, because to do so would involve in-person solicitation of legal business. (D) No, unless he has previously represented some of the arrested demonstrators on other legal matters.

(B) Aronson's actions are proper because he offered his services free of charge. Generally, a lawyer is prohibited from seeking employment by initiating personal contact with a prospective client. However, this prohibition applies only when "a significant motive" for the solicitation is the lawyer's pecuniary gain. [ABA ModelRule 7.3(a)] Thus, a lawyer who volunteers to represent someone free of charge is not subject to discipline for solicitation. (A) is incorrect because Aronson's actions are proper even if he is motivated by a desire to obtain publicity, provided that this is nota substantial motive in his offer. (C) is incorrect because, as discussed above, this situation falls within an exception to theban on in-person solicitation. (D) is incorrect because, as discussed above, it is not necessary for Aronson to have previously represented the demonstrators in order for his actions to have been proper.

Attorney Ashe is a partner in a four-partner law firm. Client Corbett entrusted $40,000 to Ashe, instructing Ashe to hold it in safekeeping for a few days and then to use it as the down payment on a piece of lakefront property. Ashe promptly deposited the money in his law firm's Law Office Account, a special bank account that the firm uses to pay the office rent, to pay staff salaries, to advance litigation expenses on behalf of clients, and the like. A few days later, when it was time to make the down payment ,Ashe discovered that one of his law partners had made two large withdrawals from the Law Office Account, reducing the account balance far below the $40,000 needed for Corbett's down payment. Ashe was unable to come up with other money to make up the difference, and Corbett thus lost the chance to buy the lakefront property. Is Ashe subject to civil liability to Corbett for mishandling the money? (A) No, because it was Ashe's law partner, not Ashe, who made the two large withdrawals that made it impossible to come up with the down payment. (B) Yes, because Ashe mishandled the money and is therefore civilly liable to Corbett for breach of his fiduciary duty. (C) No, because the legal ethics rule governing safekeeping of clients' funds is for professional disciplinary purposes only; it is not intended as a standard for civil liability purposes. (D) Yes, because a lawyer is strictly liable to a client for harm incident to the disappearance of money that the client has entrusted to the lawyer.

(B) Ashe is subject to civil liability for mishandling Corbett's money because he breached his fiduciary duty to Corbett. When Ashe received the $40,000 from Corbett, he should have deposited it in a client trust account, not in the account that the law firm uses for office expenses. Ashe's failure to put the money in the correct account was a breach of fiduciary duty that can result in both professional discipline under ABA Model Rule 1.15(a) and in civil liability under the principles expressed in the Restatement. [See Restatement §49; seealso Lurz v. Panek, 527 N.E.2d 663 (Ill.1988)—lawyer civilly liable for loss client suffers from lawyer's delay in paying money over to client] (A) is incorrect because Ashe remains liable even though it was Ashe's law partner who withdrew the money from the law firm account. [See, e.g., Blackmon v. Hale, 1 Cal. 3d 548(1970)—lawyer civilly liable when former law partner converted client's funds] (C) is incorrect because a lawyer's breach of fiduciary duty can result in both professional discipline and civil liability to a client who suffered loss as a result. (D) is incorrect because the legal standard is breach of fiduciary duty, not strict liability.

From 2000 to 2003, attorney Arentt was a partner in the firm of Able & Aman. During that period, Arnett represented client Cobb in obtaining a business loan from a bank. Cobb disclosed to Arnett a great deal of confidential information about his business and his personal assets. No other lawyer in Able & Aman gained access to that confidential information. In late 2003, Arnett died. In 2004, Carlton asked attorney Able (the senior partner of Able & Aman) to represent him in a civil suit for serious personal injuries Carlton suffered when he was run over by a delivery truck driven by one ofCobb's employees. Would it be proper for Able to represent Carlton? (A) Yes, because the information obtained by Arnett about Cobb's assets has no effect on liability in a personal injury suit. (B) Yes, because neither Able nor any other lawyer in Able & Aman gained access to Cobb's confidential information. (C) No, because the conflict created by Arnett's work for Cobb is imputed to Able. (D) No, unless Able gets Cobb's informed, written consent.

(B) It would be proper for Able to representCarlton because neither Able nor any other lawyer in Able & Aman gained access toCobb's confidential information. Even though Cobb's bank loan is not substantially related to Carlton's personal injury suit, the confidential information that Arnett got from Cobb may well become important in Carlton's suit. Knowing the extent and nature of Cobb's assets could be of great value to counsel for Carlton in advisingCarlton whether to settle or in collecting on a judgment against Cobb. Here, however, Arnett was the only lawyer who gained access to Cobb's financial information, and Arnett is now dead. Under ABA Model Rule 1.10(b), Able may represent Carlton, because neither he nor any other lawyer remaining in the Able & Aman firm had access to Cobb's confidential information. (A) is wrong because it ignores the issue presented by the confidential information. (C) is wrong because ABA Model Rule1.10(b) creates an exception to the ordinary rule that confidential information gained by one lawyer in a firm is deemed to be known by all lawyers in the firm. (D) is wrong because ABA Model Rule 1.10(b)allows Able to serve even without the consent of Cobb.

Judge Jones is a loyal member of the alumni association of Heathmoor, the women's college from which she was graduated. The 25th reunion of her graduating class is coming up next June, and she has been asked to participate in some activities designed to raise money for a gift from the class to the college scholarship fund. Which of the following activities would be proper for Judge Jones to do? I. Make a substantial personal donation to the class gift fund. II. Telephone other members of her graduating class and urge them to make a donation to the class gift fund. III. Serve on the scholarship fund committee, which devises the various fundraising strategies. IV. Be the guest of honor at a dinner to raise funds for the class gift. V. Attend a fund-raising dinner for the class gift. (A) I. and V. only. (B) I., III., and V. only. (C) I., IV., and V. only. (D) I., II., IV., and V. only.

(B) Item I. is proper because Judge Jones, like anyone else, may contribute to any cause she likes. Item II. is not proper because a judge may not personally participate in the solicitation of funds or other fund-raising activities. [CJC 4C(3)(b)] Item III. is proper because a judge may assist an organization in planning fund-raising, although the judge may not actually participate in the fund-raising activity. [CJC4C(3)(b)(i)] A judge must not be a speaker or guest of honor at an organization's fund-raising event, but mere attendance at such an event is permissible. [CJC 4C(3)(b),Commentary] Thus, item IV. is improper and item V. is proper.

Justice Jacobs was on the Supreme Court of State Beta. State Beta's Supreme Court Rules provide that in capital punishment cases, any one justice of the supreme court is empowered to grant a stay of execution pending appeal to the supreme court. Justice Jacobs granted such a stay in the case of People of State Beta v. Dillon, on the ground that Dillon had been denied the effective assistance of counsel at his trial. A few months later, Justice Jacobs retired from the supreme court and went back to private law practice. In due course, the supreme court heard the appeal in the Dillon case, rejected Dillon's effective assistance of counsel contention, and affirmed the death penalty. Acting as an indigent in propria persona, Dillon then commenced a federal habeas corpus proceeding in the United States District Court for the Eastern District of State Beta, and asked that court to appoint a private lawyer to represent him in the habeas corpus proceeding. The district court appointed Jacobs to represent Dillon. A key issue in the habeas corpus proceeding is whether Dillon was deprived of the effective assistance of counsel at his trial. MAY Jacobs represent Dillon without getting the consent of all parties to the habeas corpus proceeding? (A) No, because there is reasonable ground to doubt Jacobs's impartiality in the matter. (B) No, because when Jacobs was a supreme court justice he granted a stay of execution to Dillon. (C) Yes, because Jacobs was appointed by the district court, and his prior involvement in the matter is not sufficient grounds for refusing the appointment. (D) Yes, because the respondent in the habeas corpus case is the prison warden, not thePeople of State Beta.

(B) Jacobs may not represent Dillon in the habeas proceeding because, while serving as a supreme court justice, Jacobs grantedDillon a stay of execution. A lawyer must not represent a client in a "matter" in which the lawyer earlier participated "personally and substantially" as a judge. [ABA ModelRule 1.12(a)] The habeas corpus proceeding and the earlier appeal should be regarded as the same matter because the habeas corpus proceeding will doubtless raise many of the same issues that were decided on the earlier appeal. (The effective assistance of counsel issue is one example.) The stay of execution should be regarded as personal, substantial participation. (A) is wrong because it confuses the roles of judge and lawyer; a judge is expected to be impartial, but a lawyer is expected to be a partisan. (C) is wrong because one of the proper reasons for turning down a court appointment is that it would require the lawyer to violate a disciplinary rule. [ABA Model Rule 6.2(a)](D) is wrong because it elevates form over substance; the similarity of legal issues, not the case caption, should be determinative here.

Judge Jackson is a trial court judge who hears primarily civil tort and contract cases. Judge Jackson's mother, Martha, owns a small business. Martha Wishes to sell the business and retire to Florida. Martha and a prospective buyer come to terms on the sale, and the buyer has her lawyer draw up a sales contract. When Martha receives her copy of the contract, she is shocked to see that it is 20 pages long and full of what she considers to be legal gobbledygook. Martha realizes that she needs a lawyer to review the document. Martha does not see any reason why she should have to pay a lawyer when her own son, a judge no less, could review the document for free. Martha asks Judge Jackson to review the contract. Judge Jackson agrees. Judge Jackson marks up the contract, drafts an important rider to be attached, and returns it to Martha to present to the buyer's attorney. Were Judge Jackson's actions proper? (A) Yes, unless the buyer is likely to appear in Judge Jackson's court in the future. (B) Yes, because he did not charge Martha a fee. (C) No, because a full-time judge is not permitted to practice law. (D) No, unless his identity was not disclosed to the buyer or the buyer's attorney.

(B) Judge Jackson's actions were proper because he did not charge Martha a fee. Although a full-time judge may not practice law, there is an exception for this typeof transaction. A judge may, without compensation, give legal advice to, and draft and review documents for, a member of the judge's family. [CJC 4G] (A) is wrong because whether the buyer might appear in Judge Jackson's court does not affect Judge Jackson's ability to prepare documents for his mother. If the buyer does appear in his court, Judge Jackson's participation in that proceeding will be evaluated at that time in light of the facts. (C) is wrong because, as stated above, there is an exception for reviewing documents for relatives without compensation. (D) is wrong because, although Judge Jackson cannot act as a negotiator, there is no requirement that his identity as the person who made the revisions be kept a secret.

Entertainment Lawyer Labrie has for many years represented country music star Spangles Truhart. One evening, Labrie and Truhart were having a quiet business dinner together at a well-known restaurant. A brutish drunk, Duke Sirosis, lurched up to their table and in a loud voice began a vulgar and defamatory tirade against Truhart. Everyone in the restaurant heard the vile names Sirosis called Truhart. While all of the defamatory comments about Truhart involved her personal life, about which Labrie had no real knowledge, he felt that they could not possibly be true. At Truhart's request, Labrie commenced a slander suit against Sirosis. In his answer to the complaint, Sirosis admitted making the allegedly slanderous statements, and as an affirmative defense, he alleged that the statements were entirely truthful. When the case comes to trial, would it be proper for Labrie to act as Truhart's trial counsel? (A) Yes, but only if Truhart gives informed, written consent. (B) Yes, because Labrie is not a necessary witness. (C) No, because there is a possibility that Labrie may be called as a witness. (D) No, unless he associates and prepares co-counsel to take over in the event he is called as a witness.

(B) Labrie may act as Truhart's trial counsel because he is not a necessary witness.[ABA Model Rule 3.7(a)] A roomful of witnesses heard Sirosis's comments and could testify to them. Moreover, Sirosis has made a judicial admission that he made the statements; thus, no testimony is required on that point. Labrie has no knowledge as to the truth of the statements, as he knows nothing of Truhart's personal life; thus, he would have no relevant testimony on that issue. Therefore, Labrie is neither a "necessary" witness, nor a witness that "ought" to be called. (A) is wrong because there is no need for informed, written consent in this situation. (C) is wrong because even if there is a remote possibility that Labrie might be called, he is not a necessary witness, and it is unlikely that he would be called by Sirosis's lawyer because he could not have anything favorable or relevant to add. A mere remote possibility that the lawyer will be called as a witness is not sufficient to disqualify the lawyer from representing a client. (D) is wrong because associating and preparing co-counsel is not a prerequisite for an attorney taking a case when he is not a necessary witness and merely witnessed an event that is the subject of litigation.

Lawyer Li represented Weaver in a court proceeding to raise the alimony and child support payments set in the decree that divorced Weaver from her ex-husband, Hyde. Hyde stubbornly refused to get a lawyer in the matter. The evening before the court hearing, Hyde telephoned Li at home and asked Li to explain the legal standard the judge would apply to Weaver's request for increased payments. Li responded: "Mr. Hyde, I am not your lawyer, and I cannot give you legal advice. I think that you ought to get a lawyer in this matter, and if you need time to do that, I will ask the judge to postpone the hearing for a couple of weeks." Hyde said he didn't want a lawyer, and then asked Li whether Weaver and the children really needed more money to live on. Li responded: "Mr. Hyde, I have no personal interest to serve here--I am simply trying to do what is best for you, and your ex-wile, and your kids. Now if you really want my opinion, I'd say yes, you should pay the extra money because they do need it to live on." Hyde thanked Li and hung up. Was Li's handling of the matter proper? (A) No, because as Weaver's lawyer, Li should not have communicated directly with Hyde at all. (B) No, because Li pretended to be disinterest-ed and advised Hyde to pay the extra money. (C) Yes, because Li advised Hyde that Li was not his lawyer, that Hyde should retain one, and that Li could not give him legal advice. (D) Yes, because Li only stated his opinion and did not purport to give Hyde advice.

(B) Li's conduct was not proper because Li pretended to be disinterested and advisedHyde to pay the extra money. When deal-ing on behalf of a client with a person whois not represented by counsel, a lawyer must not state or imply that the lawyer is disinterested, and the lawyer must not give that person advice (other than advice to secure counsel) if the lawyer knows that the person's interests may conflict with those of the client. Clearly, Hyde's interests do conflict with Weaver's interests. [ABA Model Rule 4.3] Here, Li advised Hyde that he ought to pay increased alimony and child support. He compounded the problem by pretending to be disinterested, a direct violation of Rule 4.3. (A) is wrong because it is over broad; it was proper, for example, for Li to advise Hyde to get a lawyer and to offer to postpone the hearing. Moreover, if an adversary refuses to retain counsel, a lawyer must communicate directly with that person. (C) is wrong because making those statements does not exempt Li from those provisions that prohibit implying disinterest and giving advice. (D) is wrong because Li did give advice to Hyde—the advice to pay the extra money that Weaver was asking for.

Lawyer Liu volunteers her legal services every Tuesday night on the Lake County Free Legal Advice Hotline. The hotline is run under the auspices of the Lake County Superior Court, and it supplies free legal advice by telephone to callers who could not otherwise obtain legal services. Every caller assents to a "Statement ofUnderstanding" at the outset of the call, consenting to the limited nature of the legal services the hotline provides. Eighty-five lawyers volunteer their services as Liu does. They come to the hotline office at various times on various days, and the volunteers hardly ever see or talk with each other, except at the hotline' s annual chicken and beans barbecue. The nature of the hotline's work makes it impossible for the lawyers to conduct the kind of conflict-of-interest checks that an ordinary law firm would conduct before taking on a new client. One Tuesday night, Liu counseled a distraught mother about her husband's physical and mental abuse of their school-age children. Liu told the mother how to seek help from the Lake County Child Protective Services and the local law school's Domestic Violence Clinic. On the following Friday evening, a different hotline volunteer, attorney Ashcraft, counseled the mother's husband about how to prevent theLake County Child Protective Services from scooping up his children and putting them in a foster home. Because the hotline does not do conflict-of-interest checks, Ashcraft had no way to know that Liu had counseled the mother a few nights before. Which of the following is correct? (A) The judges of the Lake County Superior Court are subject to discipline for permitting the hotline to operate under their auspice without making a conflict-of-interest check before dispensing legal advice to a new client. (B) Liu, Ashcraft, and the judges of the Lake County Superior Court all acted properly because the hotline cannot do the kind of conflict-of-interest checking that a private law firm would do. (C) Liu acted properly in advising the mother, but Ashcraft is subject to discipline for giving legal advice to the father on the same subject as Liu's advice to the mother. (D) Neither the mother nor the father was a "client" of the respective lawyers who advised them. Therefore, there was no conflict of interest, and both Liu and Ashcraft acted properly.

(B) The Lake County Free Legal AdviceHotline is the kind of operation envisioned in ABA Model Rule 6.5. Under ABAModel Rule 6.5, walk-in legal clinics, advice-only clinics, legal advice hot lines, and the like, are not held to the high conflict-of-interest standards that govern ordinary law offices. Legal hotlines, walk-in clinics, and similar providers of quick legal service typically operate under conditions that make it difficult or impossible to conduct ordinary conflict-of-interest checks. Under ABA Model Rule 6.5(b), Ashcraft would be subject to discipline only if he actually knew that Liu had previously counseled the mother of the abused children. A lawyer's actual knowledge can be inferred from the circumstances [ABA Model Rule 1.0(f)], but the question does not mention any circumstances from which an inference could be made that Ashcraft had actual knowledge. (A) is wrong because it fails to account forABA Model Rule 6.5. (C) is wrong be-cause, absent actual knowledge of a conflict, the rule of imputed disqualification does not apply between two lawyers in a quick-legal-service program. [See Comment 4 to ABA Model Rule 6.5] (D) is wrong because both the mother and father were "clients" of the respective lawyers who advised them. [Comment 1 to ABAModel Rule 6.5] This is important because ABA Model Rule 6.5 loosens only the conflict-of-interest rules, not other aspects of the lawyer-client relationship such as the duty of competence, the duty of diligence, and the duty of confidentiality.

Lawyers Abner, Baker and Clark formed a law partnership; each contributed $100,000 in capital to get the firm started. Their partnership agreement provided that when a partner dies, the firm will make certain payments to the dead partner's named beneficiary. Sometime later, Clark died, leaving his daughter Clara, a doctor, as his sole beneficiary. Under the partnership agreement, the firm plans to make the following payments to Clara: $100,000, which represents Clark's share of the firm's assets, as measured by his capital contribution; $45,000, which represents Clark's share of fees that had been earned but not collected from clients at Clark's death; and a $125,000 death benefit, representing a percentage of Clark's earnings the year prior to his death and payable in monthly installments. Which of the following payments may the firm make to Clara? (A) $100,000 for Clark's share of the firm's assets. (B) $270,000, which includes Clark's share of the firm's assets, Clark's share of uncollected fees, and the death benefit. (C) $170,000, which represents the death benefit and Clark's share of uncollected fees. (D) $145,000 for Clark's share of the firm's assets and Clark's share of uncollected fees.

(B) The firm may pay all of the money as planned. Even though Clara is a non lawyer, the firm may make certain kinds of payments to her from moneys that were originally earned as legal fees. [ABA ModelRule 5.4(a)(1)] The $100,000 is a proper payment because it reflects Clark's share of the capital assets of the firm. The $45,000is a proper payment because the fees it represents had been earned, albeit not collected, at the time Clark died. The $125,000 is a proper payment because it isa reasonably computed death benefit payable over a reasonable period of time.

Attorney Abrahamson recently opened his solo law practice in Crystal Springs. His practice is fairly evenly divided between civil litigation and criminal defense. The Crystal Springs Superior Court has just appointed him to represent defendants Denton and Drews, who will be tried jointly for their alleged kidnapping and brutal murder of nine Crystal Springs school children. For which of the following reasons may Abrahamson decline the appointment? I. He believes that to represent Denton and Drews will take so much time away from his newly opened practice as to impose an unreasonable financial burden on him. II. He believes that Denton coerced Drews into helping kidnap and kill the children. III. He believes many of his potential clients in Crystal Springs will be outraged if he defends Denton and Drews. IV. He believes that confidential information he received when representing one of the prosecution's key witnesses will be useful in impeaching that witness's credibility. (A) I., II., III., and IV. (B) II. and IV. only. (C) I., II., and IV. only. (D) II. only.

(C) A lawyer can be disciplined for trying to avoid a court appointment without good cause. [ABA Model Rule 6.2] Item I. is a legitimate reason for declining an appointment. A lawyer is permitted to turn down a court appointment if it "is likely to result in an unreasonable financial burden." [ABAModel Rule 6.2(b)] Item II. is also a legitimate basis for declining appointment be-cause a lawyer may turn down a court appointment if it is likely to cause the lawyer to violate a rule of professional conduct. [ABA Model Rule 6.2(a)] IfDenton and Drews are to be tried jointly, and if Denton did coerce Drews into helping with the kidnapping and killing, there is a sharp conflict of interest between Denton and Drews. [See ABA Model Rule 1.7(a)] It would be an ethical violation to represent co-defendants with conflicting interests(consent will not solve the conflict); thus, Abrahamson can decline the appointment on this ground. The reason stated in item III. is not an acceptable reason for declining the appointment; a lawyer has a duty to represent his fair share of indigent or unpopular clients. [Comment 1 to ABA Model Rule6.2] Item IV. raises another conflict of interest that would justify Abrahamson in declining the appointment. Had he not gained confidential information from the prosecution's witness, he might have dis-covered that information independently and been able to use it to impeach the witness.As it stands, however, his ability to impeach is constrained by his duty not to use the confidential information to the disadvantage of the witness, his former client. [ABAModel Rule 1.9(c)(1)]

Attorney Aghai practices sports law. Her legal specialty is the representation of women professional basketball players who play for teams in the Women's National Basketball Association. When wearing her lawyer hat, Aghai represents her clients in various types of contract negotiations and in all other kinds of civil matters. Aghai also serves as sports agent for several women professional basketball players. When wearing her sports agent hat, Aghai's job is to advance her clients' careers in every way possible, such as team placement, public relations, product sponsorship, favorable media coverage, coach and teammate relations, healthy lifestyle, and the like. Aghai uses a single office for her law work and her sports agent work, but she charges her clients separately for the two types of work. For her sports agent work, she charges a flat annual fee that she negotiates with the client once a year. For her law work, she charges the client by the hour at a reasonable hourly rate. Aghai is lawyer and sports agent for two players on-the Topeka Tigers, Daphne Dobbs and Edith Elgin. Both women are excellent players, and both are well-educated, well-spoken, pleasantly photogenic, and popular with fans in the Midwest. The FirstContinental Bank of Topeka approached Aghai, expressing interest in signing up Daphne Dobbs to serve as the bank's spokeswoman in a new ad campaign. The bank said it would pay Dobbs "a high six-figure fee" if she would work for it, and not for any other financial institution, during the 18-month ad campaign. Aghai responded that Dobbs would fit the bank's needs very well, but that Edith Elgin would be an equally good fit and would do the work "for substantially less money." In due course, Aghai served as Elgin's lawyer in negotiating an 18-month exclusive sponsorship contract with the bank. Is Aghai subject to discipline? (A) No, because when Aghai acts on behalf of a client as sports agent, she need not follow the lawyer conflict of interest rules. (B) Yes, because Aghai was in essence charging twice for the same work when she attempted to serve as sports agent for both Dobbs and Elgin. (C) Yes, because Aghai does not keep her sports agent work clearly distinct from her lawyer work, and therefore she must follow the lawyer conflict of interest rules in both kinds of work. (D) No, because Aghai charged her clients separately for her sports agent work and her law work, and therefore she need not follow the legal ethics rules when doing sports agent work.

(C) ABA Model Rule 5.7 provides that when a lawyer offers another kind of service ancillary to her practice of law, and the ancillary service is provided "in circumstances that are not distinct from the lawyer's provision of legal services," the lawyer must follow the legal ethics rules in the ancillary service as well as the legal service. Here, Aghai does bill separately for her two kinds of service, but she offers both out of the same office, and the tasks she does as sports agent shade imperceptibly into the tasks she does as lawyer, as is illustrated by the exclusive sponsorship contract with the bank. Thus, Aghai must follow the lawyer conflict of interest rules when she acts as sports agent. [See Comment 10 to ABA Model Rule 5.7] When Aghai diverted the bank ad campaign from Dobbs to Elgin, she violated ABA ModelRule 1.7(a)(1) (concurrent conflict when the representation of one client will be directly adverse to another client), or at least ABA Model Rule 1.7(a)(2) (significant risk that the representation of one client will be limited by the lawyer's responsibility to another client). Incidentally, Aghai may also be subject to civil liability in a suit by Dobbs for breaching the duty of loyalty an agent owes to a principal. [See Restatement of the Law ofAgency 2d §§391, 394] (A) is wrong for the reasons stated above. (B) is wrong because Aghai is not doing the "same work" when she acts as sports agent for two different players. (D) is wrong because the separate charging arrangement is not sufficient by itself to clearly segregate Aghai's sports agent work from her lawyer work.

The State One Bar certifies specialists in nine fields of law, one of which is tax law. Attorney Ataris has not yet earned her certificate of specialization in tax law, but is working toward the goal. Ataris' ad in the Yellow Pages of the phone book states: "Alma Ataris. Specialist in tax law, a field in which the State One Bar grants Certificates of Specialization Call Today (209)795-6875" Ataris limits her practice to tax matters; she refers all other kinds of legal matters to lawyer Leacox, a solo practitioner in general practice. Leacox, in turn, refers all tax matters to Ataris. Without exception, Leacox and Ataris have followed that pattern of referrals for five or six years; they have no formal reciprocal referral agreement, but each invariably follows the pattern, expecting the other to reciprocate. Which of the following most correctly de-scribes Ataris's situation? (A) Ataris is subject to discipline for herYellow Pages ad, but not for maintaining the referral relationship with Leacox. (B) Ataris's Yellow Pages ad is proper, and so is her referral relationship with Leacox. (C) Ataris is subject to discipline for herYellow Pages ad, and her referral relation-ship with Leacox is improper because it would need to be nonexclusive, and the two lawyers would need to disclose it to referred clients. (D) Ataris's Yellow Pages ad is proper, but she is subject to discipline for maintaining the referral relationship with Leacox.

(C) ABA Model Rule 7.4(d) prohibits a lawyer from stating or implying that she is a certified specialist unless she has been certified by an appropriate organization that is clearly identified in the lawyer's communication. Ataris's Yellow Pages ad appears to have been artfully crafted to make unsophisticated readers think that Ataris has been certified by the State OneBar. Thus, the ad violates both ABA ModelRule 7.4(d) and ABA Model Rule 7.1, which prohibits misleading advertising. As for Ataris's reciprocal referral relationship with Leacox, the applicable rule is ABAModel Rule 7.2(b)(4), which permits a reciprocal referral agreement between lawyers, provided that the agreement is nonexclusive and the referred clients are told about the existence and nature of the agreement. ABA Model Rule 7.2(b)(4) has not yet been prominently interpreted, leaving one to wonder whether the relation-ship between Ataris and Leacox should be regarded as an "agreement." On the one hand, the question states that they have no"formal reciprocal referral agreement." On the other hand, in some legal contexts, a consciously reciprocal course of dealing can be the equivalent of an agreement. [See, e.g.,United States v. Container Corp. of America,393 U.S. 333 (1969)—competitors' reciprocal exchange of price data was held to be an agreement under section 1 of theSherman Antitrust Act] However, for purposes of this question, the relationship between Ataris and Leacox will be deemed an agreement under ABA Model Rule7.2(b)(4) because it is bound to influence their judgment about referrals to some degree. Here, the relationship violates the nonexclusive requirement because the two lawyers follow the pattern "without exception." Furthermore, the relationship would be proper only if Ataris tells referred clients about the relationship so that they can decide for themselves how to value the referral. One can reach the same conclusion by applying ABA Model Rule 1.7(a)(2)-Ataris's reciprocal relationship with Leacox gives Ataris a personal interest (obtaining future referrals) that is in conflict with the interest of her client (obtaining an unbiased referral). The conflict could be solved only by full disclosure and written consent of the affected client. [ABA Model Rule1.7(b)]

Attorney Agnes and Barnaby have been law partners for six years. They share a suite of offices in a large downtown office building and use the same secretarial staff. Unfortunately, Barnaby had difficulties with the IRS and was found guilty of intentionally understating his income for one tax year. Barnaby paid the taxes due, plus interest, penalties, and a $5,000 fine. He did not receive a prison sentence. The Disciplinary Committee of the State Bar Association suspended Barnaby from the practice of law for one year. Agnes took over Barnaby's clients when his suspension went into effect. Shortly before his suspension, Barnaby had negotiated a $30,000 personal injury settlement on behalf of his client, Charlotte. Two weeks after the settlement was reached, the defendant's insurer sent a $30,000 check to the law offices. By this time Barnaby's suspension had gone into effect. Agnes placed the check in the proper account and confirmed the amount of the fee with Charlotte. Agnes then promptly forwarded a $20,000 check to Charlotte and a $10,000 check to Barnaby, the latter check representing Barnaby's one-third contingent fee. Is Agnes subject to discipline? (A) Yes, because she should have held the $10,000 in the client trust account until Barnaby's suspension had ended. (B) Yes, because a lawyer is prohibited from sharing legal fees with a non lawyer. (C) No, because Barnaby earned the fee prior to his suspension. (D) No, because the $10,000 belonged to Agnes's firm, and she could do anything she wished with it, including sending it to Barnaby as a gift.

(C) Agnes is not subject to discipline because Barnaby earned the fee prior to his suspension. Despite his suspension, Barnaby is entitled to the fees he earned while he was still lawfully practicing law. It is true that a lawyer is prohibited from aiding a non-lawyer in the unauthorized practice of law, but here Barnaby is not practicing law, and Agnes is merely forwarding his previously earned fee. (A) is wrong because Barnaby need not wait until he is reinstated to collect a fee he earned prior to his suspension. (B) is wrong because Barnaby is not a non-lawyer, and Agnes is not splitting legal fees with him in any case. Agnes is merely transmitting Barnaby's own money to him. (D) is wrong because the money does not belong to Agnes, and even if it did, she cannot do anything she wishes with it. For example, there are rules prohibiting the sharing of legal fees with non lawyers.

For many years, tax attorney Aguero has handled all of the tax work for client Carrara, a famous post-modern sculptor. Carrara's large sculptures cost hundreds of thousands of dollars each and are sold mostly to wealthy collectors and museums. Carrara also, however, produces the occasional small, modestly priced work. Aguero greatly admires Carrara's talent and yearns for one of the small sculptures to display in his office. One evening, Carrara invited Aguero to his studio to discuss some tax returns that had to be filed the next day. In the studio, Aguero saw a small sculpture that would be perfect for his office. At the close of their tax discussion, Aguero told Carrara how much he admired the small sculpture and offered to buy it for $10,000, its approximate fair market value. Carrara told Aguero that it was not for sale. In due course, Aguero sent Carrara a $750 fee bill for the tax work. A few days later, the small sculpture was delivered to Aguero's office with the following note from Carrara: "My dear Aguero: I hope this small piece of my work will satisfy your recent fee bill. I want you to have it as a token of my gratitude for the excellent tax advice you have given me all these years. I hope you will enjoy having it in your office. Your friend, Carrara." Would Aguero be subject to discipline for accepting the small sculpture from Carrara? (A) Yes, because the gift is of significant monetary value. (B) Yes, because the value of the sculpture is far out of proportion to the $750 worth of work Aguero did for Carrara. (C) No, because Aguero did not solicit the gift. (D) No, unless $10,000 would be an unreasonably high legal fee for the work Aguero did for Carrara.

(C) Aguero would not be subject to discipline for accepting the sculpture because he did not solicit the gift. Although ABA Model Rule 1.8(c) prohibits a lawyer from solicit-ing a substantial gift from a client when the lawyer is not related to the client, it does not prohibit a lawyer from accepting an unsolicited gift from a client, even if the gift is substantial (although the gift may be voidable for undue influence). Moreover,Comment 6 to ABA Model Rule 1.8 states that a lawyer may accept a gift from a client if the transaction meets general standards of fairness. Here, Aguero did not solicit the gift, and there are no facts to suggest undue influence or unfairness. Thus, the gift is proper. (A) is wrong because it is too broad. A lawyer may accept a gift of substantial value from a client if the conditions stated above are satisfied. (B) and (D) are wrong because the value of Aguero's recent work for Carrara is irrelevant. Aguero did not charge more than the $750. In addition to discharging the $750 fee bill, Carrara obviously intended to make a gift to Aguero in gratitude for years of work in the past.

Attorney Alexander and her client Cardone endured a stormy attorney-client relationship until Alexander finally withdrew due to Cardone's repeated refusals to pay Alexander's fee bills. At the end of the relationship, Cardone owed Alexander more than $10,000. Cardone said he would not pay because Alexander's legal services were "defective." In a final effort to avoid having to sue Cardone for the unpaid fees, Alexander proposed a settlement agreement to Cardone. Under the proposed agreement, Alexander would accept $4,000 as full payment, reserving the right to sue Cardone for the other $6,000 if Cardone filed a State Bar disciplinary complaint against Alexander or filed a legal malpractice action against Alexander. Cardone signed the settlement agreement without consulting outside counsel, and Alexander did not suggest that he should consult outside counsel before signing it. Is Alexander subject to discipline for entering into the settlement agreement with Cardone? (A) No, because Alexander brought about an amicable settlement of the fee dispute with Cardone. (B) No, provided that there was a good faith dispute between Alexander and Cardone about the quality of Alexander's services and the amount of fees due.( C) Yes, because Alexander did not advise Cardone to seek outside counsel before entering into the settlement agreement. (D) Yes, because Alexander compromised a potential malpractice claim by contract with her client.

(C) Alexander is subject to discipline because she did not advise Cardone to seek outside counsel. A lawyer may not settle a legal malpractice claim or potential claim with an unrepresented client or former client without first advising that person in writing to seek outside legal advice about the settlement and giving the person a reason-able chance to obtain such advice. [ABAModel Rule 1.8(h)(2)] Although Cardone has apparently not made a formal claim of malpractice here, he has asserted thatAlexander's services were "defective," and that is regarded as sufficient to bring Rule1.8(h)(2) into play. The settlement agreement in this question also gives the client an incentive not to report a lawyer's mis-conduct to the bar. Some state bars have found similar settlement agreements improper because they frustrated the bar's efforts at self-regulation and could be prejudicial to the administration of justice.[See Arizona State Bar Op. 91-23 (1991)](A) is wrong because although lawyers are urged to settle fee disputes amicably[Comment 9 to ABA Model Rule 1.5], this particular settlement agreement involves both a fee dispute and a malpractice claim.(B) is wrong because the dispute about the quality of Alexander's services is what causes the problem here, as explained above. (D) is wrong because it is too broad.A lawyer may settle a malpractice claim if the claimant is independently represented or if the lawyer advises the claimant in writing that he should seek independent legal advice before entering into the settlement. [ABA Model Rule 1.8(h)(2)]

Attorney Angstrom was appointed by the court to defend client Cheever at Cheever's criminal trial for second degree murder. Angstrom started interviewing potential witnesses. When she interviewed Cheever's landlord, the landlord said that on the night of the murder, Cheever came home very late and was wearing a shirt covered with blood. The landlord died before trial without speaking to state authorities. Which of the following best states what Angstrom should do with respect to the information she has learned from the landlord? (A) Angstrom should voluntarily reveal the information to the prosecutor prior to trial because the death of the landlord has made it impossible for the prosecutor to obtain the information in any other way. (B) Angstrom should urge Cheever to allow her to reveal the information to the prosecutor, and if Cheever refuses, Angstrom should withdraw. (C) Angstrom should keep the information in confidence unless Cheever authorizes her to reveal it, even though the death of the landlord has made it impossible for the prosecutor to obtain the information other than from Angstrom. (D) Angstrom should use her own best judgment about how to treat the information; it is neither privileged nor confidential because it was not given to her by her client or by an agent of her client.

(C) Angstrom should keep the information in confidence unless Cheever authorizes her to reveal it, even though the death of the landlord has made it impossible for the prosecutor to obtain the information other than from Angstrom. Angstrom obtained this information from the landlord in the course of representing her client; therefore, it is subject to the attorney's duty of confidentiality. Absent the consent of the client, an attorney must not reveal any information relating to the representation of the client. [ABA Model Rule 1.6] (A) is incorrect because an attorney has no ethical obligation to reveal harmful facts, and in fact, may be disciplined for doing so. (B) is incorrect because, as noted above, there is no duty to reveal this information; thus, there is no obligation to urge the client to reveal the information or withdraw. (D) is incorrect because this information is confidential. The ethical duty of confidentiality covers more kinds of information than the attorney-client privilege, which covers only confidential communications between the attorney and client. The ethical duty of confidentiality covers any information the attorney obtains relating to the representation of the client, no matter what the source of the information.

Attorney Aoki is defending client Childs in a civil fraud case in which it is relevant to know what advice Childs received in confidence from an independent certified public accountant, Ben Counter. This jurisdiction has no evidentiary privilege for confidential communications between accountants and their clients. Counter telephoned Aoki and asked how he should respond to the plaintiff's lawyer's request to speak with him privately about the case. Reasonably believing that Counter would not be harmed by refusing to talk informally with the plaintiff's lawyer, Aoki responded: "If the plaintiff's lawyer subpoenas you to testify, then you must do so, but I encourage you not to talk to him about the case unless you are under subpoena." Was her advice to Counter PROPER? (A) No, because the advice Counter gaveChilds was not protected by an evidentiary privilege. (B) No, because Aoki interfered with the plaintiff's access to evidence. (C) Yes, because Counter acted as Childs's agent in rendering accounting advice toChilds. (D) Yes, because it was improper for the plaintiff's lawyer to seek a private discussion with Counter about the case.

(C) Aoki's advice to Counter was proper because Counter acted as Childs's agent in rendering accounting advice. An attorney may request that someone other than a client refrain from voluntarily giving relevant information to another party if the person isa relative or agent of the client and the attorney reasonably believes that the person's interests will not be adversely affected by refraining from giving the information.[ABA Model Rule 3.4(f)] Here, Aoki reasonably believed that refusing to talk informally with the plaintiff's counsel would not harm Counter, and Counter wasChilds's agent in rendering the accounting advice. (A) is wrong because it is Counter's agency, not privilege, that affects the propriety of Aoki's advice. The lack of an evidentiary privilege does not give the plaintiff's counsel a right to talk to Counter informally if Counter chooses not to. (B) is wrong because Aoki did not interfere with the plaintiff's access to evidence; if the plain-tiff wants to know what Counter has to say, he can simply take Counter's deposition.(D) is wrong because it was not improper for the plaintiff's counsel to attempt to talk informally with a third-party witness such as Counter.

Solo practitioners Arias and Armer share office space. Each of them has organized her practice as a professional corporation. The sign on their office door reads: "Arlene Arias, P.C. Personal Injury Law" "Alice Armer, P.C. General Practice" Arias and Armer frequently consult with each other about their respective cases, and they often refer clients to one another. Sometimes they work on cases together under a fee-sharing arrangement. When one of them is out of the office, the other responds to client inquiries to the extent that she is able to facilitate that practice, each attorney has physical access to the other's client files. Plaintiff Puenta hired Arias to sue McDougal's Bakery for personal injuries he sustained when he bit into a piece of glass in a dinner roll baked by McDougal's. McDougal's liability insurance carrier, American Assurance Associates, asked Armer to serve as defense counsel in the case. May Armer take the case? (A) Yes, but only if Arias and Armer believe that they can effectively represent their respective clients, and only if Puente, McDougal's, and American give informed, written consent. (B) Yes, because the rule of imputed disqualification does not apply to Arias and Armer. (C) No, even if Arias and Armer believe that they can effectively represent their respective clients, and even if Puente, McDougal's, and American give informed, written consent. (D) No, unless Arias and Armer believe that they can effectively represent their respective clients.

(C) Armer may not take the case even if Ariasand Armer believe that they can effectivelyrepresent their respective clients, and evenif all parties give informed, written con-sent. The key issue is whether Arias andArmer are considered a "firm" for purposes of the imputed disqualification rule with respect to conflicts of interest. Because lawyers in a firm are usually treated as a single unit for conflict of interest purposes, different lawyers in the same firm must not represent opposing parties in a civil case.[ABA Model Rule 1.10(a)] Relevant factors in determining whether lawyers who share office space are deemed a firm include whether they: hold themselves out to the public as a single unit, frequently consult and assist each other, refer cases to each other, work jointly on cases, and have access to each other's files. All of these factors are present in this case. Thus, Armer and Arias are deemed a firm, and Armer is disqualified from accepting the employment because Arias's disqualification is imputed to her. (A) is wrong because a client must not be asked to consent if a disinterested lawyer would conclude that the client should not agree to the representation. Representing both sides in litigation is such a circumstance. [ABA Model Rule 1.7,Comment 23] Moreover, ABA Model Rule1.7(b)(3) does not permit client consent to solve a conflict of interest when one client sues another client represented by the lawyer in the same proceeding. (B) is wrong because, as discussed above, the rule of imputed disqualification does apply to Arias and Armer even though they are not partners. (D) is wrong because, as discussed above, representation is improper here even if the lawyers believe that they can effectively represent the clients and even if the clients consent.

For several years, attorney Aston worked at the United States Department of Labor as part of a small group of attorneys whose responsibilities included compiling certain corporate safety records and monitoring compliance with federal regulations. Under a federal statute, factories of a certain type and size must report all work-related accidents to Aston's office. Aston's duties included compiling an annual report containing the accident statistics of all of the reporting companies. The report is used internally and in discussions with companies, but it is not distributed to the general public. A person may obtain a copy of the report, but must file a formal request under the Freedom of InformationAct procedures adopted by the Labor Department. During the last three years, Chemco has had more accidents than any of the other report-ing companies. Six months ago, Aston left theLabor Department and took a job with a private law firm. Charles comes to Aston seeking representation in a suit against Chemco for injuries he sustained last month while working at one of Chemco's factories. Although unsure as to whether he should take the case, Aston, who is just starting out in private practice and cannot afford to turn clients away, agrees to represent Charles. Is Aston subject to discipline? (A) Yes, because he obtained relevant information on Chemco while working as a government attorney. (B) Yes, unless Aston obtains the consent of the Department of Labor. (C) No, because the information is available by formal request under the Freedom ofInformation Act. (D) No, if Aston does not use the information obtained while employed as a government attorney to the material disadvantage of Chemco.

(C) Aston is not subject to discipline for taking the case because the relevant information he obtained while working as a government attorney is not confidential. The general rule is that a government lawyer who receives confidential government information about a person must not later represent a private client whose interests are adverse to that person, if the information could be used to the material disadvantage of that person. [ABA Model Rule 1.11(c)] The rule covers only "confidential" information, which means information that the government is prohibited from revealing or has a privilege not to reveal, and which is not otherwise available to the public. Here, because the information is available under the Freedom of Information Act, it is not confidential. In fact, any attorney representing Charles could obtain the information; thus, Aston is free to use it. (A) is wrong because an attorney is not barred from ever working on a case where he gained any relevant information while working for the government. To bar representation, the information must be confidential. (B) is wrong because this type of consent is required when the attorney takes on a representation in private practice in a matter in which the lawyer participated personally and substantially while in government service. A "matter" is a set of specific facts involving specific parties.Here, Aston was not involved in any matter while in government service that concerned Charles's claim against Chemco. (D) is wrong because the information is not confidential and thus can be used against Chemco. Furthermore, even if the information were confidential, mere nonuse would not be sufficient; Aston would not be permitted to represent Charles.

A series of brutal daylight muggings in downtown Sedatia brought fear to the citizens of that normally placid city. The police captured one Diablo, who was charged with the muggings and, in due course, was ordered to stand trial in Sedatia. Two days before the jury selection began, a local newspaper reporter cornered the prosecutor, District Attorney Axelrod, in the Sedatia Cafe and got her to make the following statement: I'm certain Diablo is the right man; among other things, we have discovered that he was previously convicted three times for brutal muggings in other states. Is Axelrod subject to discipline for making the statement to the reporter? (A) No, because prior criminal convictions area matter of public record. (B) No, because a lawyer has a First Amendment right to inform the public about pending cases. (C) Yes, because she should have known that the statement would be quite likely to prejudice the trial. (D) Yes, because a prosecutor must not make public comment on a pending case.

(C) Axelrod is subject to discipline because she should have known that the statement was likely to prejudice the trial. A lawyer whois connected with a case must not make a public statement outside the courtroom that the lawyer reasonably should know would have a substantial likelihood of materially prejudicing the case. [ABA Model Rule3.6(a)] Axelrod's comment to the reporter falls in that category because it revealed very damaging material that had not been, and probably would not be, admitted into evidence. A defendant's prior convictions generally are inadmissible as evidence of his conduct on the occasion in question.[See Fed. R. Evid. 404] Even if Diablo elected to waive his privilege against self-incrimination and testified on his own behalf, the prior convictions would prob-ably not be admissible to impeach him.[See Fed. R. Evid. 609(a)—mugging does not involve dishonesty or false statement, and a judge would probably exclude felony mugging convictions because of the high risk of prejudice] Note that there is an additional constraint on the prosecutor in a criminal case. A prosecutor must not make extrajudicial comments that have a substantial likelihood of heightening public condemnation of the accused. [ABA ModelRule 3.8(0] Axelrod's statement also runs afoul of this rule by revealing prior crimes that would heighten public condemnation of Diablo. (A) is wrong because even if the prior convictions are a matter of public record, that does not absolve Axelrod for making a statement she knew would substantially prejudice the case. Further-more, while in a technical sense Diablo's prior convictions in other states are matters of public record (if one knew where to go and what to look for, one could dredge them out of the court records of the other states), the prior convictions were doubtless not part of the public record in Diablo's present prosecution. (B) is wrong be cause it is too broad; lawyers do have First Amendment rights to express themselves about pending cases, but those rights are limited by the due process rights of litigants to fair court proceedings. (D) is wrong because it is also too broad; a prosecutor may make some kinds of public comment about pending cases, but not comments that are likely to cause prejudice.

Federal prosecutor Fedman is stationed in Maryland; he is gathering evidence to support federal racketeering charges against a swindler named Phatseaux. Fedman believes in good faith that a wealthy Florida banker, Bucks, has personal knowledge about three federal felonies committed by Phatseaux, but Bucks will not disclose what he knows about Phatseaux. How-ever, Fedman has recently learned from a secret informant that Bucks illegally drained off $4.7 million from a failing Florida savings and loan institution—a state felony punishable by 10years in prison. Fedman therefore mailed a letter from his office in Maryland to Bucks in Florida,s tating in relevant part: "I am coming to Florida next week. If you don't give me what I need concerning Phatseaux, I am going to tell the Florida prosecutors what you did to that savings and loan. Do you know what those old boys instate prison do with fellows like you?" Is Fedman subject to criminal liability because of his evidence-gathering technique? (A) No, because Fedman had a legal right to tell the Florida prosecutors about Bucks, and he simply warned Bucks what he intended to do if Bucks did not cooperate. (B) No, because Fedman was acting in good faith, believing that Bucks had relevant, unprivileged information that was material to an ongoing federal criminal investigation. (C) Yes, because Fedman made an interstate threat to accuse Bucks of a crime for the purpose of extracting valuable information that he could use against Phatseaux. (D) Yes, because Fedman acted under color of law to deprive Bucks of his federally protected civil rights.

(C) Fedman is subject to criminal liability because his evidence-gathering technique amounted to extortion, an interstate threat against Bucks for the purpose of extracting information to use against Phatseaux. Under modern statutory law, the crime of extortion covers obtaining anything of value, tangible or intangible, by making various kinds of threats, including a threat to accuse a person of a crime. [See, e.g., 18U.S.C. §875(d)—felony to obtain some-thing of value by transmitting in interstate commerce a threat to accuse a person of a crime; see also Perkins & Boyce, 442-52]The information that Fedman wanted fromBucks was something of value in the Phatseaux investigation, and Fedman clearly threatened to accuse Bucks of a crime if Bucks did not cooperate. Further-more, the threat was transmitted interstate between Maryland and Florida, thus bring-ing it within the scope of the federal statute cited above. (A) is incorrect because Fedman did have a legal right to tell theFlorida prosecutors about Bucks and the savings and loan, but Fedman did not have a right to threaten to do so (i.e., to commit extortion) in order to coerce Bucks to cooperate. (B) is incorrect because it ignores the law of extortion. Just as it would have been illegal for Fedman to extract the information by physical torture, it was illegal to extract it by extortion. (D)is not as good as (C) because it is vague and because the right to be free from extortion, though important, is not a federally protected civil right.

BioTek, Inc. s a small, modestly financed company that develops methods to use a medical patient's own stem cells to repair nerve damage caused by disease. BioTek obtained a very valuable United States patent on one of these methods. BioTek's wealthy competitors trembled when the patent was issued, and they joined as plaintiffs in a federal court declaratory judgment suit to have the patent declared invalid on the ground that BioTek had publicly used the patented method more than one year before the date of the patent application. If proven, that is enough to invalidate the patent under United States law. Attorney Adler represents BioTek in the litigation. Adler knows, because of a privileged conversation with BioTek's Director of Research, that only one person in the world knows for sure when the patented method was first publicly used. That one person is Elmo Wintz, Ph.D., a bright but reclusive research biochemist who spends every waking hour in his lab at BioTek. Which of the following would be proper for Adler to do to prevent the plaintiffs from learning what Dr. Wintz knows? I. Fail to tell counsel for the plaintiffs that they should take Dr. Wintz's deposition. II. Advise BioTek to grant Dr. Wintz a two-year sabbatical leave, at full pay, on a remote tropical island in the South Pacific. III. When the plaintiffs pose an interrogatory asking for the identity of witnesses who know when the patented method was first publicly used, advise BioTek to answer,"We lack certain knowledge of any such witness." IV. At Dr. Wintz's deposition, object to any question touching on the date the patented method was first publicly used, and instruct Dr. Wintz not to answer the question. (A) All of the above. (B) None of the above. (C) I. only. (D) I., III., and IV. only.

(C) Item I. is correct. The adversary system assumes that the parties will gather the relevant evidence competitively. [Comment 1 to ABA Model Rule 3.4] Adler does not have a duty to voluntarily tell opposing counsel where the golden egg is located. On the other hand, if opposing counsel asks in a proper discovery request,"Where is the golden egg?" then counsel must provide an honest response. [ABAModel Rules 3.3(a)(1), 3.4 (a) - (c)] Item II.is false. Adler must not unlawfully obstruct the opposition's access to evidence [ABAModel Rule 3.4(a)], and if he knowingly violates the rules of discovery, he will be subject to both professional discipline and litigation sanction. [ABA Model Rule3.4(c); Fed. R. Civ. P. 37] Item III. is false.Because the term "certain" has at least eight meanings, the quoted answer to the interrogatory is either incomplete or affirmatively misleading. When advising his client on how to respond to an interrogatory, Adler must not recommend a misleading, incomplete, or evasive response; doing that is treated the same as failing to respond.[Fed. R. Civ. P. 37(a)(2)(B)(3); ABAModel Rule 3.4(c)—knowing disregard of an obligation under the rules of a tribunal]Item IV. is false. At Dr. Wintz's deposition, the only time Adler may advise Dr. Wintz not to answer a question is (i) to preserve a privilege, (ii) to enforce a court-ordered limit on discovery, or (iii) to giveAdler a chance to move for a protective order. [See Fed. R. Civ. P. 30(d)(1)] The question does not mention anything to indicate that Dr. Wintz's knowledge is privileged, that the court has previously imposed a limit on discovery, or that a protective order would be appropriate.Therefore, Adler will be subject to both discipline and litigation sanction if he instructs Dr. Wintz not to answer questions about the date of first public use. [Id.; ABAModel Rule 3.4(c)]

Bar applicant Lingenfelter is applying to become a member of the Bar of State A. The application questionnaire asks whether she has ever used any narcotic in violation of State A law. When she was a high school student in State A, Lingenfelter occasionally smoked marijuana, which is a minor misdemeanor under State A law. The statute of limitations has long since run on those offenses. Lingenfelter is convinced that she could not validly be kept out of the State A bar for those offenses. She therefore believes that the question is irrelevant and an invasion of her privacy in violation of Stale A's Constitution. She fears, however, that challenging the question could brand her as a troublemaker and delay her admission to the bar. Which of the following would be proper? I. Answer the question in the negative, without saying more. II. Answer the question in the affirmative, but explain the circumstances. III. Decline to answer the question, citing the invasion of privacy provision of the State A Constitution. IV. Decline to answer the question, citing the federal constitutional privilege against self-incrimination. (A) I., II., III., and IV. (B) None of the above. (C) II. and III. only. (D) II., III., and IV. only.

(C) Item I. would not be proper because a bar applicant must not make untrue statements on a bar application. [ABA Model Rule8.1(a)] Item II. would be proper, because it is truthful; it may, of course, provoke a further inquiry by the bar. Item HI. would be proper because it openly challenges the validity of the question on legally tenable grounds. [See ABA Model Rule 3.4(c)]Item IV. would not be proper because the self-incrimination privilege is in applicable here. Even if the privilege against self-incrimination applies to questions on bar applications (a debatable proposition), it does not apply when criminal punishment is barred by the statute of limitations. [SeeHazard & Hodes, §62.6]

Lawyer Loomis is the head of the in-house law department of Darlington KiddieWear Corp., which has its principal place of business in State A. Under the law of State A, it is a felony to manufacture or sell children's sleepwear that is not fire retardant. The president of Darlington informed Loomis in confidence that Darlington is stuck with a whole warehouse full of children's pajama fabric that does not meet State A's fire standards, and that to avoid financial disaster Darlington will use the fabric to make children's sleepwear and take its chances on legal liability. Loomis was unable to convince the president to change his mind; she then raised the issue with Darlington's board of directors, which ratified the president's decision. Will Loomis be subject to discipline if she resigns as house counsel and reports the matter to the appropriate State A law enforcement authorities? (A) Yes, because she is required to preserve the corporation's confidential information even after she resigns. (B) Yes, because there is no adequate reason for permissive withdrawal on these facts. (C) No, because she is entitled to reveal this type of confidential information. (D) No, because her duty to preserve confidential information ceases with her resignation as house counsel.

(C) Lawyer Loomis will not be subject to discipline for reporting the matter to the appropriate authorities because she is entitled to reveal this type of confidential information. Loomis may resign her inhouse counsel position because Darlington's board insists on following a course of action that is both repugnant and criminal. [ABAModel Rule 1.16(b)(2), (4)] The children's sleepwear is likely to cause substantial bodily harm or even death. Because an attorney is entitled to reveal confidential information to the extent she reasonably believes necessary to prevent reasonably certain death or substantial bodily harm, Loomis may report this matter to the appropriate authorities. [ABA Model Rule1.6(b)(1)] In addition, if the highest authority for an organization fails to take appropriate action regarding a violation of law, then a lawyer for the organization may report the relevant information to an appropriate person outside of the organization, if the lawyer reasonably believes that reporting is necessary to prevent substantial injury to the organization. This is true even if the information would otherwise be protected by the duty of confidentiality(which is not the case here because the sale of the fabric is likely to cause substantial bodily harm). [ABA Model Rule 1.13(c)]Here, the sale of the fabric is a violation of law, and selling the fabric could result insubstantial injury to the organization if the fabric catches fire. (A) is wrong because Loomis is entitled to reveal confidential information to prevent reasonably certain death or substantial bodily harm or to prevent substantial injury to the organization. (B) is wrong because the repugnance and criminality of the proposed conduct are both sufficient grounds for permissive withdrawal. [ABA Model Rule 1.16(2),(4)] (D) is wrong because the duty of confidentiality continues even after the termination of the attorney-client relation-ship. [See Comment 18 to ABA ModelRule 1.6]

Lawyer Lederlee was assigned by the court to defend an indigent person, former college English teacher Deniew, at her trial for the murder of her husband. The jury convicted Deniew, and she was sentenced to 40 years in prison. Lederlee's court appointment expired at the end of the trial, but he promised Deniew that he would represent her without cost in taking an appeal from her conviction. Lederlee advanced $350 on Deniew's behalf to cover the expenses of the appeal, knowing that Deniew would probably not be able to pay him back. While the appeal was pending, Deniew wrote the manuscript for a book about life in a women's prison. She hired Lederlee to negotiate a contract with a publisher to have the book published, and in return for the contract work, she promised to pay Lederlee 30% of the royalties from her book. Is Lederlee subject to discipline? (A) Yes, because he entered into a literary rights contract with his client while her appeal was still pending. (B) Yes, because he advanced appeal expenses for his client, knowing that she probably could not pay him back. (C) No, unless 30% of the book royalties is unreasonably high for the contract negotiation work. (D) No, even if 30% of the book royalties is unreasonably high for the contract negotiation work.

(C) Lederlee is not subject to discipline unless30% of the book royalties is unreasonably high for the contract negotiation work.Here, the lawyer has, in essence, agreed to negotiate the publication contract in return for a contingent fee. Like all other fees, a contingent fee is subject to the general requirement of reasonableness. [ABAModel Rule 1.5] (A) is wrong because this is not the kind of literary rights contract that is prohibited. A lawyer must not acquire literary or media rights to a story concerning the lawyer's representation of a client until after the legal matter is entirely concluded. [ABA Model Rule 1.8(d)] Here, the client is the author, and the book is about her life in prison, not about her case or her lawyer's representation of her. A lawyer may represent a client in a transaction concerning literary property in which the lawyer's fee consists of a share of the ownership of the property, provided that the arrangement complies with the general rules about attorneys' fees and does not give the lawyer a proprietary interest in the subject of litigation. [Comment 9 to ABAModel Rule 1.8] (B) is wrong because a lawyer may advance litigation expenses fora client, even though he is aware that she probably cannot pay him back. ABAModel Rule 1.8(e)(2) permits the lawyer simply to pay the litigation expenses for an indigent client, even without the pretense of calling it an advance. (D) is wrong because all lawyer fees are subject to the general requirement of reasonableness.

Client Cristin sought the advice of lawyer Leona on a difficult and sensitive family problem. Cristin suspected that her husband had been molesting Daisy, Cristin's 12-year-old daughter by a prior marriage. Cristin asked Leona what she should do. Leona advised Cristin that all three members of the family should consult Frances, a licensed family counselor who specializes in precisely this sort of problem. Fearing that if Cristin were aware of the law she would not seek counseling, Leona purposely failed to tell Cristin that a new state statute requires family counselors to report to the district attorney all instances of suspected child abuse. Cristin and her family consulted Frances, and Frances reported the matter to the district attorney, as she was required to do by law. The district attorney commenced criminal proceedings against Cristin's husband, much against the wishes of both Cristin and Daisy. Were Leona's actions proper? (A) Yes, if Leona believed in good faith thatCristin would not seek counseling if she knew about the statute. (B) Yes, because it is the policy of the state that all instances of child abuse be reported to the appropriate authorities. (C) No, because a lawyer should fully advise a client of relevant information. (D) No, unless Leona believed in good faith that the daughter had actually been molested.

(C) Leona's actions were not proper because a lawyer should fully advise a client of all relevant information, particularly when the lawyer has reason to believe that the information would be regarded as important by the client. The lawyer should furnish the client with all the information that is necessary to allow the client to participate intelligently in making deci-sions about the matter. [ABA Model Rule1.4] Here, the existence of the statute was a fact necessary for Cristin to make an intelligent decision about how to proceed.(A) is wrong because it is not Leona's place to withhold information because she believes she knows what is best for Cristin.Cristin is entitled to all relevant information. (B) is wrong because the state policy does not absolve Leona from her ethical duty to keep her client informed. (D) is wrong because, again, withholding information is not an option regardless of whether Leona thought she was doing the right thing.

Lawyer Leroy is interested in obtaining legal business from the United Smelter and Mining Employees Union ("USMEU"), which has many organized workers in the state. As a result of a recent mine fire and explosion in which several miners were killed, USMEU has succeeded in persuading the appropriate state agency to bring an administrative action against the mining company for failing to install smoke detectors, which might have saved some lives in the mine disaster. Although Leroy is in no way involved in the case, he sees this as an opportunity to get future business from the union by showing USMEU that he is strongly on their side in the mine disaster case. Leroy telephones a popular call-in radio show, and makes the following statement: My name is Leroy and I am an attorney. I'd just like to say that I am shocked and appalled at the callousness of the mining company that caused the recent disaster in which so many miners were killed. From what I have read, it appears to me that the mining company was willful and wanton in its failure to install smoke detectors. I am behind the union 100%; I hope the company will not be allowed to escape the consequences of this despicable conduct. Without Leroy's knowledge or consent, his statement was later printed in several newspapers in the state. Is Leroy subject to discipline for his conduct? (A) Yes, because he was substantially motivated by his desire to attract fee-paying business. (B) Yes, because lawyers must not make public comments concerning pending litigation. (C) No, because he did not make any false or misleading claims about himself or his services. (D) No, because the statement was printed in the newspapers without his knowledge or consent.

(C) Leroy is not subject to discipline because he did not make any false or misleading claims about himself or his services.Lawyers, like other citizens, have the right to express their views in the media on newsworthy issues. Even if a lawyer's sole purpose in seeking media publicity is to lure clients, the state may not impose professional discipline on the lawyer absent a compelling state interest. A lawyer who uses the media to lure clients may, however, be disciplined for making statements or claims that are false or misleading about the lawyer or his services. [ABAModel Rule 7.1] Here, Leroy made no statements about himself or his services other than the fact that he is a lawyer and his opinion about the incident. There is nothing false or misleading in his communication. (A) is wrong because, as discussed above, the fact that Leroy was motivated by the desire to attract fee-paying business is irrelevant. This is not a case of in-person or live electronic solicitation, which is the only circumstance in which this consideration is relevant. (B) is wrong because it is over broad. Lawyers can and do make public statements about pending litigation all the time. Lawyers who are involved in a proceeding cannot make statements that they know will have a substantial likelihood of materially prejudicing an adjudicative proceeding. [ABAModel Rule 3.6(a)] That is not the case here; Leroy has no reason to believe his opinion will materially prejudice the state agency bringing the action. (D) is wrong because the fact that the statement was printed in the newspapers is of absolutely no consequence; it does not affect the propriety of Leroy's behavior regardless of whether he gave consent.

Client Chandler, a concerned environmentalist, hired lawyer Lipscomb to obtain preliminary and permanent injunctions against a highway construction project that would require draining and filling certain wetlands inhabited by migratory waterfowl. Lipscomb is the nation's leading expert in wetland preservation law, and he charges $400 per hour for his services. Chandler agreed to pay him at that rate. She gave him a $40,000 advance on attorney fees and a $5,000 advance to cover future litigation expenses. Lipscomb deposited the entire $45,000 in his client trust account. Lipscomb then spent 80 hours preparing and filing a complaint and preparing and arguing a motion for a preliminary injunction. He paid a court filing fee of $50, plus $1,950 in witness fees to wetlands experts who testified at the hearing on the preliminary injunction motion. The judge denied the preliminary injunction motion. Lipscomb sent Chandler a bill for $32,000 in attorney fees and $2,000 in litigation expenses, and he told her he would deduct those sums from the advances she had given him unless he heard from her to the contrary within 15 days. In light of the loss of the preliminary injunction motion, Chandler was outraged at the size of Lipscomb's fee; she immediately fired him and demanded the prompt refund of her entire $45,000. Which of the following amounts must Lipscomb promptly refund to Chandler? (A) $0 (B) $43,000 (C) $11,000 (D) $13,000

(C) Lipscomb must promptly refund $11,000because that amount is not in dispute.Lipscomb claimed $32,000 in fees (80hours at $400 per hour) from the $40,000 fee advance (leaving an excess of $8,000). Lipscomb further claimed $2,000 from the$5,000 advance for expenses (leaving an excess of $3,000). Adding $8,000 + $3,000produces an $11,000 refund due immediately from Lipscomb to Chandler. [ABAModel Rule 1.16(d)] Chandler apparently disputes Lipscomb's right to $32,000 in fees; thus, that disputed amount must remain in Lipscomb's client trust account until the fee dispute is settled. [ABA ModelRule 1.15(c)] (A) is wrong because when a lawyer is fired or withdraws, he must immediately refund the unspent portion of the expense advance and the portion of the fee advance that he does not claim to have earned. [ABA Model Rule 1.16(d)] (B) is wrong because Lipscomb is entitled to retain the disputed $32,000 in his client trust account until the fee dispute is settled.(D) is wrong because Lipscomb may retain the disputed portion of the expense ad-vance, as explained above.

Lawyer Lockwood represents defendant Downs in a drug smuggling case. Downs is in pretrial custody in a distant city and cannot be reached by telephone. One key issue in Downs's case is on the cutting edge of search and seizure law, and Lockwood believes that he needs help to deal with the issue competently. Lockwood's former law professor is a nationally known expert on search and seizure law. Lockwood calls his professor to ask lot his help, and also asks that the professor keep this information confidential. To frame the issue accurately, Lockwood tells the law professor some information that Downs revealed to Lockwood in confidence. Lockwood does not tell the professor tile name of his client. Is Lockwood subject to discipline for disclosing Downs's confidential information to the professor? (A) Yes, if the professor was not licensed to practice in that jurisdiction. (B) Yes, unless Downs had specifically authorized Lockwood to make such a disclosure. (C) No, because the disclosure was necessary to effectively carry out the representation. (D) No, because Lockwood did not reveal his client's name.

(C) Lockwood is not subject to discipline for disclosing Downs's confidential information to the professor because the disclosure was necessary to effectively carry out the representation. Unless a client has specifically instructed the lawyer to the contrary, that lawyer can reasonably assume that he has implied authority from the client to disclose confidential information when necessary to carry out the representation. That is particularly true in cases such as this one where the lawyer cannot easily communicate with his client. [ABA Model Rule1.6(a)] (A) is wrong because a lawyer may seek advice from an expert without the expert being licensed in the jurisdiction; and in any case, it does not affect the confidentiality rules. (B) is wrong because specific authorization is not required; it can be implied. (D) is wrong because refraining from revealing the client's name is not sufficient to permit revelation of confidential information. There must be an exception to the confidentiality rules or authorization by the client.

United Consumers Bank operates a "Professional Referral Hotline" for its depositors. Any United depositor who needs to find a physician, lawyer, accountant, dentist, or the like, can telephone the hotline and obtain a free referral from lists of professionals compiled by United. The lists are limited to professionals who maintain an average balance of at least $10,000 in a time deposit account at United, but the professional does not pay a fee to United for receiving a particular referral. Lawyer Lomax keeps $10,000 on deposit with United for the express purpose of being included on its lawyer referral list. Is this arrangement proper? (A) Yes, because United is functioning in the role of a lawyer referral service. (B) Yes, because neither United's depositors nor the professionals pay a fee for referrals. (C) No, because Lomax is required to keep$10,000 on deposit to be included on the list. (D) No, because this arrangement constitutes an association with a non lawyer for the practice of law.

(C) The arrangement is not proper because Lomax is required to keep $10,000 on deposit to be included on the list. A lawyer may not give "anything of value" to a person for recommending the lawyer's services. [ABA Model Rule 7.2(b)] The bank benefits in many ways by increasing the amount of its deposits; for example, its deposits determine how much it can lend to borrowers. Thus, obtaining deposits from lawyers is of value to the bank, and that is one reason it has devised the referral scheme.(A) is wrong because although a lawyer may pay the usual charges of a not-for-profit or qualified lawyer referral service [ABAModel Rule 7.2(b)], banks operate for profit, and there is no indication that the bank has been approved by the appropriate regulatory authority as a qualified lawyer referral service. (B) is wrong because Lomax is giving something of value for the referrals, as explained above, even though there is no fee for individual referrals. (D) is wrong because this arrangement does not constitute an improper partnership or association with a non lawyer for the purpose of practicing law. A lawyer's professional association with a non lawyer is improper if the non lawyer: (i) owns an interest in the practice; (ii) is an officer or director of a business involving law practice; or (iii) has the right to control the lawyer's professional judgment.[ABA Model Rule 5.4(d)] None of these is the case here; United is acting solely as a referral agent, and has nothing to do with the operation of the lawyer's practice.

Continuously since 1910, the law firm of Hardwicke & Chandler has practiced under that name in State A. The founders of the firm,Horace Hardwicke and Carlisle Chandler, are long dead. No partner named Chandler now practices with the firm. Horace Hardwicke IV(the great-grandson of the founder) is presently the managing partner of the firm. Five years ago, Hanna Hardwicke, the daughter of Horace Hardwicke IV, became a partner in the firm. Hanna recently left law practice to take a life tenure appointment on the State A Supreme Court. May the firm continue to use the name Hardwicke & Chandler? (A) No, because no partner named Chandler now practices with the firm. (B) No, because Hanna ceased private practice to enter public service. (C) Yes, unless the firm name would be misleading. (D) Yes, even if the firm name will mislead some prospective clients.

(C) The firm may continue to use the name Hardwicke & Chandler if it is not mislead-ing. ABA Model Rule 7.5(a) permits a firm to practice under a trade name, provided that the trade name is not misleading in violation of ABA Model Rule 7.1. (A) is wrong because a firm may continue using the name of a deceased partner. [Comment1 to ABA Model Rule 7.5] (B) is wrong because Hanna Hardwicke is not a name partner. Generally, when a name partner enters public service and is not in private practice for a substantial period, the firm must cease using that person's name. [ABAModel Rule 7.5(c)] Hanna Hardwicke's elevation to the State A Supreme Court does not by itself require the firm to cease using the name Hardwicke because that surname refers to her great-great-grandfather. If ,however, the use of the Hardwicke name in the firm name would mislead potential clients (e.g., by making them think they could gain an advantage in the State A Supreme Court by hiring that firm), then continued use of the name would violate the ethics rules. [See ABA Model Rule 7.1] (D) is wrong because the rules on firm names are subject to the more general provisions on misleading communications.

Robbins & Hood Chemical Corp. is a family owned corporation, the shares of which have never been offered to the public. It is not subject to the jurisdiction of the Securities and Exchange Commission. It sells a variety of chemicals throughout the United States. The United States antitrust laws prohibit competitors from agreeing among themselves about the prices they will charge their customers(a practice called horizontal price fixing). The board of directors of R&H long ago adopted a rule that all employees must obey the anti trust laws, and that anyone who does not will be fired and will lose all company benefits. R&H's in-house General Counsel got wind of a rumor that some R&H sales people on the West Coast had met with their counterparts from competing companies and had reached an agreement that, without too much of a stretch, could be called horizontal price fixing. After consulting with R&H's chief executive officer ("CEO"), the General Counsel hired an outside antitrust lawyer, Leona Leopard, to investigate the situation, advise the company on what to do, and defend the company if it is sued for antitrust violations. Leopard and her young associate, Arthur Athol, investigated and found that R&H's West Coast sales employees had indeed met with competitors about prices, thus exposing R&H to possible criminal liability, not to mention millions of dollars worth of civil treble-damage suits. Leopard reported these findings and her antitrust advice in a confidential letter addressed jointly to the General Counsel and the CEO. The General Counsel wrote back, thanking Leopard for her work and asking her to stand by to defend the company if needed. Months went by, and Leopard heard nothing more. Athol, the young associate, grew restless; without telling Leopard, he told a friend in the United States Justice Department what R&H sales people had done. The Justice Department began a price fixing investigation of R&H and its competitors. Which of the following is correct? (A) Leopard is subject to discipline for failing to report R&H's situation to the antitrust enforcement authorities in the Justice Department, but Athol's conduct was proper. (B) The conduct of both Leopard and Athol was proper, and neither of them will be subject to civil liability if R&H sues them for legal malpractice. (C) Athol is subject to discipline for tipping off the Justice Department, but Leopard's conduct was proper unless she knew what Athol was about to do or ratified it later, failed to rectify it when she could have, or failed to train and supervise Athol properly. (D) Neither Leopard nor Athol is subject to discipline, but both of them may be subject to civil liability if R&H sues them for legal malpractice.

(C) This question is governed by ABA ModelRule 1.13, not by the SEC's regulations under the Sarbanes-Oxley Act, becauseR&H is not publicly owned and is not subject to the jurisdiction of the SEC. Athol is subject to discipline for tipping off the Justice Department because he violated the duty of confidentiality imposed byABA Model Rule 1.6(a). When Leopard and Athol investigated the price fixing rumor at the request of R&H's General Counsel, they were operating under ABAModel Rule 1.13(d), which applies to lawyers who are hired "to investigate an alleged violation of law" or to "defend an organization" or its people against a claim arising out of an alleged violation of law.That means that ABA Model Rule 1.13(c)does not apply, and they must not report to outsiders about what they find. The passage in (C) about Leopard's conduct correctly states the standard expressed inABA Model Rule 5.1(c) about the duties of a supervising lawyer. Both parts of (A) are wrong—Leopard acted properly, and Athol is subject to discipline, as explained above.(B) is wrong because Athol's conduct was not proper. Moreover, Athol probably committed legal malpractice when he tipped off the Justice Department, andLeopard might be vicariously liable for his malpractice because she was his supervisor. (D) is wrong because Athol is subject to discipline for violating ABA ModelRule 1.6(a).

Client Carson, a self-employed furnace repairman with no assets, was run over in a pedestrian crosswalk by a moving van driven by an employee of Deluxe Transport and Storage Corporation. Carson's injuries were so severe that he could not work, or even seek work, for a period of 18 months after the accident. Carson asked lawyer Lucas to represent him in a personal injury action against Deluxe. At Carson's request, Lucas agreed to represent Carson on a contingent fee basis. Carson requested that Lucas provide him with certain financial assistance during the pendency of the lawsuit. Which of the following may Lucas do? I. Lend Carson $5,000, pursuant to an agreement reviewed by independent counsel, to be used to support Carson's family during the pendency of the suit. II. Lend Carson $10,500, pursuant to an agreement reviewed by independent counsel, to pay for Carson's medical treatment. III. Advance Carson $1250 for court-filing~ fees and deposition-reporter fees, subject to repayment by Carson when the case is concluded. IV. Advance Carson $2,000 to pay the expert witness fee of Dr. Stromberg, a medical expert consulted solely for testimony and not for treatment. Carson promises to repay the money when the case is concluded. (A) I., II., and III. only. (B) II. and III. only. (C) II., III., and IV. only. (D) III. and IV. only.

(D) A lawyer may advance court costs and litigation expenses on the client's behalf. A lawyer is subject to discipline, however, for rendering any other kind of financial assistance to a client in the context of contemplated or pending litigation. [ABA ModelRule 1.8(e)] Items III. and IV. represent court costs and litigation expenses, which are permissible. Family support is not an expense of litigation; thus item I. is improper. Similarly, treatment of Carson's injuries is not an expense of litigation, therefore item II. is also improper.

Gerard Goop is a pimp; he makes a handsome living by arranging tricks for a group of prostitutes and protecting them from harm. As a sideline, he also distributes high-quality heroin to his prostitutes and to a few carefully selected cash customers. When one of Goop's prostitutes was arrested for prostitution, Goop asked youngsolo practitioner Prentice to defend her at her trial. Goop explained the nature of his business to Prentice so that Prentice would understand why it was important to defend the young woman vigorously and successfully. Goop offered Prentice a "down payment" on the legal fee, consisting of a shoeboz full of crisp, new $100 bills. Prentice was uncertain about getting involved with Goop, and especially about accepting a large sum in cash from Goop. Prentice told Goop that he would think about Goop's request and would let him know later that day. When Goop left the office, Prentice telephoned his friend and mentor, retired judge Judson. Prentice told Judson the entire story, including Goop's name, and asked Judson whether it would be ethical to defend the prostitute and accept Goop's cash. Is Prentice subject to discipline for telling Judson the whole story? (A) Yes, because the attorney-client privilege forbids Prentice from revealing what Goop told Prentice in confidence. (B) No, because Prentice's prospective client was the accused prostitute, not Goop. (C) Yes, but it would have been proper ifPrentice had not used Goop's name. (D) No, because the ethical duty of confidentiality has an exception that allows a lawyer to reveal confidential information to obtain legal ethics advice.

(D) ABA Model Rule 1.6(b)(4) and Comment9 explain that a lawyer may reveal information that would otherwise be confidential if the lawyer's purpose is to obtain legal advice about complying with the legal ethics rules. (A) is wrong because the applicable doctrine here is the ethical duty of confidentiality, not the attorney-client privilege, and the ethical duty contains the exception described above. (B) is wrong because the exception to the ethical duty would apply in this situation no matter whether Goop is regarded as the client, or the prostitute as the client, or both of themas joint clients. (C) is wrong because the exception to the ethical duty would apply in this situation, whether or not Prentice identified Goop by name. As a practical matter, however, a lawyer who discloses confidential information to obtain legal ethics advice may wish to couch the information hypothetically, in order to minimize the chance of harm to the client. [See Comment 4 to ABA Model Rule 1.6]

Young associate Aster was assisting senior partner Parker in writing the reply brief in an appeal for one of Parker's clients. In doing the legal research, Aster discovered a recent case from the controlling jurisdiction that had not been cited in the adversary's brief. In Aster's opinion, the case was directly opposed to the position of Parker's client. Aster asked Parker about citing it in the reply brief, but Parker explained that, in his view, the case was not directly on point and did not have to be cited. Aster and Parker argued back and forth at some length and finally decided to submit the question to one of the other senior partners in the firm for a fresh view. That partner sided with Parker, and the reply brief was filed without mentioning the case. Should Aster write a short letter to the appellate court and the adversary lawyer, explaining his position and enclosing a copy of the case? (A) Yes, because Aster had a duty to call the case to the court's attention. (B) Yes, because Aster must not allow another person to interfere with his professional judgment. (C) No, because Aster must not communicate with a court ex parte about the merits of a pending case. (D) No, because Aster should abide by Parker's resolution of the matter.

(D) Aster should abide by Parker's resolution of the matter. A subordinate lawyer does not violate the Rules of ProfessionalConduct by acting in accordance with a supervisor's reasonable resolution of an arguable question of professional duty.[ABA Model Rule 5.2(b)] Here, it seems clear that the question was arguable be-cause the third attorney called in to deter-mine the relevance of the case also felt it was not on point. (A) is wrong becauseAster only has a duty to call the case to the court's attention if the case is directly on point. That is a debatable question, andAster's supervisors have determined the case is not directly on point. Thus, Aster need not reveal the case. (B) is wrong because this is not the situation intended to be addressed by the rule against allowing a third party to influence the lawyer's judgment, which usually arises when a third party pays the lawyer's fees to represent another. Of course, a subordinate lawyer should be influenced by his supervisor.That is not an excuse for clearly unethical conduct, but on a debatable issue, such as the one presented here, the subordinate lawyer is free to defer to the supervisor's judgment. (C) is wrong because in most jurisdictions a lawyer may communicate in writing with the court about the merits of a pending case if he sends a copy to opposing counsel. This communication is not considered ex parte. [See Restatement§113, comment c]

For the past 40 years, solo practitioner Febell has practiced municipal bond law in State A. Because he is nearing retirement age, Febell takes in young attorney Spryte as a partner. Their partnership agreement provides that Febell will train Spryte in municipal bond law, that Febell will receive 75% of the partnership's net earnings during the first three years, and that Spryte will receive the remaining 25%. The agreement further provides that if Spryte leaves the partnership before the end of the first three years, he will remit to Febell 75% of all fees he earns thereafter from municipal bond work he does in State A. Finally, the agreement provides that if Febell and Spryte are still partners when Febell retires, Spryte will pay Febell retirement benefits of $3,000 per month until Febell's death; in return, upon his retirement, Febell will turn over to Spryte all of the partnership assets (including goodwill) and will not thereafter practice municipal bond law in State A. Are Febell and Spryte subject to discipline for entering into this partnership agreement? (A) No, because the agreement gives Febell retirement payments in return for the restriction on his right to practice. (B) No, because the agreement enables Febell to sell the partnership assets in return for the restriction on his right to practice. (C) Yes, because of the restriction on both Febell's and Spryte's right to practice. (D) Yes, because of the restriction on Spryte's right to practice if he leaves the partnership within the first three years.

(D) Febell and Spryte are subject to discipline for entering into the partnership agreement because of the restriction on Spryte's right to practice. A lawyer must neither make nor offer a partnership agreement that restricts a lawyer's right to practice after termination of the relationship, except for an agreement concerning benefits upon retirement. [ABA Model Rule 5.6(a)]Under this agreement, if Spryte leaves the partnership within the first three years, he must pay Febell 75% of the fees he earns thereafter from municipal bond work he does in State A. This provision unduly restricts Spryte's right to practice. (A) is wrong because it overlooks the improper restriction on Spryte's right to practice. (B)is wrong because it overlooks the restriction on Spryte. Furthermore, it implies that the sale of assets in exchange for the restriction on Febell is necessary to restrict Febell's right to practice; it is not. (C) is wrong because the restriction on Febell's right to practice is properly incident to the retirement benefits he will receive from Spryte.

Inventor Inovacio asked patent lawyer Patton to represent him in obtaining a U.S. patent on a new computer technique for predicting the growth patterns of tumors in the human body. Patton informed Inovacio that he had never worked on that kind of patent application before, and that he would have to do extensive background research on the patentability of computer techniques. Patton will be able to use the knowledge that he gains through the research to serve other clients who wish to obtain patents for all manner of other computer techniques. Patton offered to do the work for Inovacio for his standard hourly rate, but Inovacio proposed instead to assign Patton a 10% interest in the patent, if and when it was issued. Patton agreed to do the work on that basis, and he and Inovacio entered into an appropriate written fee agreement. Patton did the work; the patent was ultimately issued and proved so valuable that Patton was able to sell his 10% interest for $9.7 million. Is Patton subject to discipline? (A) Yes, because he acquired a proprietary interest in the subject of the representation. (B) Yes, because it is unreasonable to charge one client for background research that will be used to earn fees from other clients. (C) No, because Inovacio agreed to the fee arrangement. (D) No, unless $9.7 million is an unreasonably high fee for the work that Patton did.

(D) If $9.7 million is within the bounds of reason as a fee for the work Patton did, he is not subject to discipline. [See ABAModel Rule 1.5(a)] Among the various factors that point to the reasonableness of Patton's fee are: the novelty and difficulty of the patentability issue, the fact that Inovacio was the one who suggested the fee arrangement after having been offered a standard hourly fee, the value of the result that Patton obtained for Inovacio, and the contingent nature of the arrangement which imposed a high risk on Patton. (A) is wrong because a lawyer is prohibited from acquiring a proprietary interest in the subject of litigation he is conducting [ABAModel Rule 1.8(1)], and obtaining a UnitedStates patent is not litigation. Even if this were a litigation case, the contingent fee exception to the rule would apply. [ABAModel Rule 1.8(i)(2)] (B) is wrong because a fee that is otherwise reasonable does not become unreasonable simply because the lawyer can use the knowledge gained to earn fees from other clients. (C) is wrong because the mere fact that the client agreed to the fee arrangement does not by itself make the fee reasonable. Many factors, including the time, labor, and skill required to do the job, are considered in determining whether the fee is reasonable. [ABA ModelRule 1.5(a)]

Judge Jardon is a full-time trial judge in State A. State A has a statute that prohibits employment discrimination against gays and lesbians. In addition to her judicial work, Judge Jardon is the chief executive officer of a corporation that is closely held by Judge Jardon and her three brothers. The corporation owns and operates a nursing home in State A. Because of strong anti-homosexual religious beliefs on the part of residents, the nursing home does not employ gays and lesbians. Is it proper for Judge Jardon to continue as chief executive officer of the corporation? (A) Yes, because a judge is prohibited from association with an organization that practices invidious discrimination on the basis of race, sex, religion, or national origin only. (B) Yes, unless the management of the family-owned business takes so much time that it interferes with the judge's judicial duties. (C) No, because a judge is not allowed to serve as an officer, director, manager, general partner, advisor, or employee of a business entity. (D) No, because the nursing home practices employment discrimination against gays and lesbians.

(D) It is not proper for Judge Jardon to continue as chief executive officer of the corporation because the nursing home practices employment discrimination in violation of state law. [CJC 2C] While CJC 2C concerns only organizations that discriminate based on race, sex, religion, or national origin, the Commentary to that rule states that a judge's membership in an organization that discriminates on other grounds in violation of local law would violate CJC 2and 2A, which relates to the appearance of impropriety and the judge's duty to comply with the law. (A) is incorrect because the nursing home's discrimination violates state law and is thus improper. (B) is incorrect because, as discussed above, the judge's remaining an officer in the corporation would be improper even if the time it took did not interfere with her judicial duties. (C) is incorrect because it is too broad; CJC 4D(3)(a) allows a judge to be involved with a business that is closely held by the judge or her family.

Stork, Inc. is a large private adoption agency that handles over 65% of all private adoptions in State Blue. Stork provides each set of adoptive parents with a list entitled "Approved List of Lawyers Handling Private Adoptions." Stork's representatives tell prospective adoptive parents that it is in their best interest to obtain counsel who has experience in adoptions. Stork has checked out every lawyer on its list to make sure that the lawyer had experience in private adoptions as well as a reputation for honesty and ethical behavior. Because of Stork's volume of business, State Blue attorneys recognize the advantages of being placed on the "Approved List." After an attorney desiring placement on the list has been screened by Stork, the attorney is required by Stork to sign a form agreement before the lawyer's name is placed on the list. To assure the adoptive parents that any lawyer on the list would follow through with the adoption to its conclusion, the required form agreement contains the statement, "I agree that under no circumstances will I withdraw from any case where I have been retained by parents adopting through Stork." Attorney Ash has handled many private adoptions and is highly regarded as an honest and competent attorney. He would like to be placed on Stork's list. Would it be proper for Ash to have himself included in the list? (A) Yes, because the Approved List is a valuable service, and people often do not know where to find a lawyer qualified to handle a matter such as a private adoption. (B) Yes, provided Lenny does not give any-thing of value to Stork in exchange for being placed on the list. (C) No, because Stork is acting as Lenny's agent, and a lawyer may not use an agent to contact prospective clients in a manner that would be unethical solicitation if done by the lawyer. (D) No, because the form agreement allows a third party to exercise influence over the lawyer-client relationship.

(D) It would not be proper for Lenny to have his name included on Stork's list because the form agreement he is required to sign allows a third party to exercise influence over the lawyer-client relationship. A lawyer must not allow a person who re-commends, employs, or pays her for serving a client to direct or regulate the lawyer's professional judgment. [ABAModel Rule 5.4(c)] Stork is, in effect, recommending the lawyer, and the restriction on withdrawal in the form agreement clearly interferes with the lawyer's professional judgment. (A) is wrong because the good intentions behind the list do not remove the interference with the lawyer's professional judgment. Likewise, (B) is wrong because, even if Lenny does not give Stork anything of value, the arrangement is still improper because of the restriction on withdrawal. (C) is wrong because the list does not amount to improper solicitation.

Judge Jamagian sits on a State A trial court. Every six years, State A trial judges must stand as candidates in a public election to determine whether they will retain their positions. Judge Jamagian will be a retention candidate in the election to be held nine months from now. In that same election, her husband, attorney Ali, will be a candidate for Lieutenant Governor of State A. Which of the following may Judge Jamagian do? I. Establish a campaign committee that will immediately begin soliciting reasonable contributions for Judge Jamagian's campaign. II. Allow her name to be listed on Republican Party election materials, along with the name of her husband and other Republican candidates for elective offices. III. Publicly endorse her husband as a candidate for Lieutenant Governor. IV. Attend political gatherings in the company of her husband, and speak on behalf of both herself and him. V. Personally solicit contributions to her own campaign. VI. Personally solicit contributions to her husband's campaign. (A) I., II., and IV. only. (B) I., II., and III. only. (C) II., IV., V., and VI. only. (D) I. and II. only.

(D) Item I. is proper; the judge's campaign committee may begin soliciting contributions one year before the election [CJC5C(2)], and her retention election is nine months away. Item II. is proper; the judge may allow her name to be listed on a party ticket or other election materials along with the names of other party candidates.[CJC 5C(3)] Item III. is improper; a judge may not publicly endorse a candidate for a public office, and the Code contains no exception to that prohibition. [CJC5A(1)(b)] Item IV. is improper; the judge may attend a political gathering in the company of her husband, and she may speak on her own behalf, but she must not speak on his behalf. [CJC 5A(1)(b),5C(1)(b)(0] Item V. is improper; a judge is prohibited from personally soliciting contributions for her own campaign. [CJC5C(2)] Item VI. is improper; a judge is also prohibited from personally soliciting contributions for her spouse's campaign.[CJC 5A(1)(e)]

Attorney Ambrose is admitted to practice only in State A, where he specializes in securities and real estate finance law. In that role, Ambrose advised his client, Corbuster, that the law of State B did not require Corbuster to include information about certain mineral rights in a disclosure statement that Corbuster had to file in State B in order to sell some real estate limited partnership interests to State B citizens. Acting on Ambrose's advice, Corbuster did not disclose the information and did sell partnership interests to State B citizens. Later, Ambrose became a full-time trial court judge in State A Later still, State B brought a criminal action against Corbuster for failing to disclose the mineral rights information in his State B disclosure statement. One of Corbuster's defenses is that he lacked the necessary criminal intent because he was acting in good faith based on the advice of his counsel, Ambrose. Corbuster needs Ambrose's testimony to prove that Ambrose did indeed advise him that he was not required to disclose the mineral rights information. Ambrose, in State A, is beyond the subpoena power of the State B court. May Ambrose voluntarily testify on behalf of Corbuster? (A) No, because judges are disqualified from serving as witnesses in criminal cases. (B) No, because he is not admitted to practice in State B, and his testimony about State B law would be inadmissible. (C) Yes, because a judge may testify as a witness, except in his own court or one under its appellate jurisdiction. (D) Yes, because his testimony would concern the giving of the advice, not his client's character.

(D) Judge Ambrose may testify because he is testifying to facts, not the defendant's character. CJC 2B prohibits a judge from testifying voluntarily as a character wit-ness, but it says nothing about serving as an ordinary fact witness. (A) is wrong because there is no such rule. Judges are not disqualified from testifying in criminal cases. (B) is wrong because Ambrose did not need to be admitted to practice in StateB in order to advise Corbuster about StateB law. Even if that were untrue, Ambrose's testimony would still be admissible as evidence of Corbuster's lack of criminal intent. (C) is wrong because it states a nonexistent rule. While a judge is not competent to be a witness at a trial over which he himself is presiding [see Fed. R.Evid. 605], no rule forbids a judge from serving as an ordinary fact witness in a case that is pending before a different judge on his own court or a court that is under his court's appellate jurisdiction.

Worker Workman sued his employer, Drexel Moving and Storage Co., claiming that he was permanently and totally disabled due to a back injury he suffered on the job. Lawyer Lenhart represented Drexel in the case. Lenhart strongly suspected, but had no proof, that Workman continued his hobby of skydiving after the alleged back injury. In due course, Lenhart met with Workman's lawyer for a settlement discussion. Lenhart told Workman's Lawyer: "We won't give you a dime on this claim; we've got movies of your guy jumping out of an airplane two weeks after his phoney injury." Workman's lawyer excused herself to make a telephone call to Workman. When she asked Workmail whether he had been skydiving after the accident, he admitted that he had. With the consent of their' respective clients, the two lawyers then settled the case for $400. Is Lenhart subject to discipline? (A) No, because bluffing is an accepted tactic in settlement negotiations between lawyers. (B) No, because Lenhart's bluff successfully unmasked a fraudulent claim. (C) Yes, because it was improper to pay $400to settle a fraudulent claim. (D) Yes, because Lenhart lied about having movies.

(D) Lenhart is subject to discipline because he lied about having movies. When dealing on behalf of a client with a third person, a lawyer must not knowingly make a false statement of law or material fact. [ABAModel Rule 4.1(a)] Lenhart knew that he had no movies; his statement to Workman's lawyer was a bald lie. (A) is wrong because Lenhart's statement was a knowing misrepresentation of material fact, not the kind of puffery that is tolerated in settlement negotiations. [See Comment 2 to ABA Model Rule 4.1] (B) is wrong because the ends do not justify the means. (C) is wrong because Drexel consented to the $400settlement. Given. Drexel's risk of going to trial in a case where the plaintiff is claiming total and permanent disability due to a back injury, the small settlement was not unreasonable.

Public defender Purdum was assigned to represent defendant Dewitt at Dewitt's preliminary hearing on a charge of kidnapping for ransom. Against Purdum's advice, Dewitt testified on his own behalf at the preliminary hearing. Dewitt was bound over for trial. At that point, Dewitt's elder brother provided money to hire a private lawyer to defend Dewitt, and public defender Purdum was discharged. Dewitt testified on his own behalf at the trial, and the jury acquitted him. Later, in connection with his work on another matter, Purdum read the transcript of Dewitt's trial. Based on information Purdum learned while representing Dewitt, Purdum concluded that Dewitt had committed perjury, both at the preliminary hearing and at the trial. May Purdum reveal Dewitt's perjury? (A) Yes, but only with respect to the perjury at Dewitt's preliminary hearing. (B) Yes, Purdum may reveal both instances of perjury if Dewitt refuses to recant. (C) No, unless Purdum believes Dewitt is dangerous. (D) No, because disclosure would violate Purdum's duty of confidentiality.

(D) Purdum may not reveal Dewitt's perjury because to do so would violate Purdum's duty of confidentiality. No exceptions to the confidentiality requirement apply to these facts. There is no indication that revealing the perjury is necessary to prevent reason-ably certain death or substantial bodily harm. Also, there is no indication thatDewitt's perjury is a crime that is reasonably certain to result in substantial injury to the financial interests of another, in furtherance of which Dewitt has used Purdum's services. The obligation to reveal perjury under the Model Rules does not apply because that obligation ceases at the end of the proceedings, and both proceedings here have concluded. (A) is wrong because the proceeding has ended and, therefore, Purdum is obligated to keep the information in confidence.(B) is wrong for the same reason. The prerequisite of urging the client to recant is relevant while the proceeding is in progress, but not after. Furthermore, even if the trial was ongoing, Purdum would not be obligated to reveal the perjury because he was not representing Dewitt at trial. Purdum represented Dewitt in the preliminary hearing, which has concluded, ending his obligation to disclose. (C) is wrong because what Purdum believes about Dewitt's dangerousness is not relevant. There is no exception to the duty of confidentiality based on the client's violent propensities.


Set pelajaran terkait

Peritoneum, Mesentery and Omenta

View Set

Prueba sobre el cortometraje Lexxicon 2 Diez Minutos

View Set

Chapter 11: Managing Successful Products, Services, and Brands

View Set